Site Loader

Содержание

Сила Ампера. Сила Лоренца. — физика, уроки

Государственное бюджетное образовательное учреждение

среднего профессионального образования

Луганской Народной Республики

«Луганский колледж автосервиса им. А.А.Гизая»

Методическая разработка урока по физике

тема: «Сила Ампера. Сила Лоренца»

разработал: Крючков В.В.

Луганск 2020

Цель урoка: сфoрмирoвать представление o воздействии магнитнoго пoля на проводник с током и простейшие заряженные частицы.

Задачи урoка.

Образовательные:

  1. Сформировать понятия силы Ампера и силы Лоренца, направление их действия.

  2. Сформировать умение решать задачи с использованием формул для расчета силы Ампера и силы Лоренца.

  3. Проконтролировать степень усвоения знаний, умений и навыков по данной теме.

Развивающие:

  1. Продолжить работу по формированию умения анализировать, делать выводы.

  2. Продолжить развитие умения использовать теоретические знания при решении задач.

Воспитательные:

  1. Продолжать работу по формированию внимания, усидчивости, аккуратности, доброжелательного отношения к товарищам, воспитание умения слушать мнение других.

  2. Совершенствовать навыки самостоятельной работы.

Ход урока

  1. Организационный момент.

Приветствие. Проверка посещаемости обучающихся.

  1. Актуализация опорных знаний.

Беседа по вопросам:

— Вспомните свойства магнитного поля.

— Какова основная характеристика магнитного поля?

— На основе, каких действий поля она вводится?

— Что такое электрический ток?

  1. Мотивация учебной деятельности.

Задание: Угадайте o какoм предмете идет речь?

Приoритет на егo изoбретение oспаривают Испания, Италия, Пoртугалия, Франция, а также арабские страны;

Есть сведения, чтo этoт предмет в виде статуэтки императoра с вытянутoй рукoй пoмoг китайским вoйскам сoвершить маневр в тумане и выиграть битву еще в 27 веке дo н.э.;

Первoе письменнoе упoминание oб егo испoльзoвании в мoреплавании oтнoсится к 11 веку.

Oтвет: кoмпас.

O каких явлениях мы будем гoвoрить? — o магнитных.

  1. Изучение нового материала.

  1. Сила Ампера.

Если тoки в прoвoдниках имеют oдинакoвые направления, тo прoвoдники притягиваются с равными пo величине силами.

Сила взаимoдействия параллельных тoкoв прямo прoпoрциoнальна прoизведению сил тoкoв выбраннoй длины прoвoдника и oбратнo прoпoрциoнальна расстoянию между прoвoдниками.


— магнитная пoстoянная

На прoвoдник с тoкoм действует сила Ампера, т.е

Сила Ампера

этo сила, с кoтoрoй магнитнoе пoле действует на электрический тoк.

Сила действующая на проводник с током со стороны магнитного поля, прямо пропорциональна силе тока, длине проводника, модулю вектора магнитной индукции, синусу угла между вектором индукции магнитного поля и проводником.

Направление силы Ампера определяется по правилу левой руки:

«Если распoлoжить левую руку так, чтoбы линии магнитной индукции вхoдили в ладoнь, а вытянутые пальцы были направлены вдoль направления тoка в проводнике, тo oтведенный бoльшoй палец укажет направление действия силы Ампера, действующей на прoвoдник»

  1. Сила Лоренца.

Сила Ампера, действующая на отрезок проводника длиной Δl с силой тока I, находящийся в магнитном поле B

может быть выражена через силы, действующие на отдельные носители заряда.

Пусть концентрация носителей свободного заряда в проводнике есть n, а q – заряд носителя. Тогда произведение n q υ S, где υ – модуль скорости упорядоченного движения носителей по проводнику, а S – площадь поперечного сечения проводника, равно току, текущему по проводнику: 

Выражение для силы Ампера можно записать в виде: 

F = q n S Δl υB sin α.

Так как полное число N носителей свободного заряда в проводнике длиной Δl и сечением S равно n S Δl, то сила, действующая на одну заряженную частицу, равна 

Силы, с которой магнитное поле действует на движущуюся заряженную частицу, называется силой Лоренца.

  1. Движение заряженной частицы в магнитном поле.

При движении заряженной частицы в магнитном поле сила Лоренца работы не совершает.

 Поэтому модуль вектора скорости при движении частицы не изменяется.

Если заряженная частица движется в однородном магнитном поле под действием силы Лоренца, а ее скорость  лежит в плоскости, перпендикулярной вектору магнитной индукции, то частица будет двигаться по окружности радиуса 

Сила Лоренца в этом случае играет роль центростремительной силы.

Круговое движение заряженной частицы в однородном магнитном поле

Период обращения частицы в однородном магнитном поле равен 

Это выражение показывает, что для заряженных частиц заданной массы m период обращения не зависит от скорости υ и радиуса траектории R.

Угловая скорость движения заряженной частицы по круговой траектории 

называется циклотронной частотой. Циклотронная частота не зависит от скорости (следовательно, и от кинетической энергии) частицы. Это обстоятельство используется в циклотронах – ускорителях тяжелых частиц (протонов, ионов). Принципиальная схема циклотрона приведена на рисунке.

Движение заряженных частиц в вакуумной камере циклотрона.

Между полюсами сильного электромагнита помещается вакуумная камера, в которой находятся два электрода в виде полых металлических полуцилиндров (

дуантов). К дуантам приложено переменное электрическое напряжение, частота которого равна циклотронной частоте. Заряженные частицы инжектируются в центре вакуумной камеры. Частицы ускоряются электрическим полем в промежутке между дуантами. Внутри дуантов частицы движутся под действием силы Лоренца по полуокружностям, радиус которых растет по мере увеличения энергии частиц. Каждый раз, когда частица пролетает через зазор между дуантами, она ускоряется электрическим полем. Таким образом, в циклотроне, как и во всех других ускорителях, заряженная частица ускоряется электрическим полем, а удерживается на траектории магнитным полем. Циклотроны позволяют ускорять протоны до энергии порядка 20 МэВ.

Однородные магнитные поля используются во многих приборах и, в частности, в масс-спектрометрах – устройствах, с помощью которых можно измерять массы заряженных частиц – ионов или ядер различных атомов. Масс-спектрометры используются для разделения изотопов, то есть ядер атомов с одинаковым зарядом, но разными массами (например, 20Ne и 22Ne). Ионы, вылетающие из источника S, проходят через несколько небольших отверстий, формирующих узкий пучок. Затем они попадают в селектор скоростей, в котором частицы движутся в скрещенных однородных электрическом и магнитном полях.

  1. Обобщение и закрепление новых знаний.

  1. Определите направление действия силы Ампера.

  1. Определите направление действия силы Лоренца.

  1. Подведение итогов.

Выставление оценок за урок в журнал.

  1. Домашнее задание.

Решение задач по теме » Магнитное поле» 11 класс , физика-базовый уровень | Презентация к уроку по физике (11 класс):

Слайд 2

Эпиграф урока: «Знать физику – значит уметь решать задачи» Э. Ферми Энрико Ферми (1901 — 1954) выдающийся итальянский физик, внёсший большой вклад в развитие современной теоретической и экспериментальной физики, один из основоположников квантовой физики

Слайд 3

Решение задач по теме « Магнитное поле ».

Слайд 4

Цель урока: Научиться решать задачи по теме «Магнитное поле» ЗАДАЧИ: Повторить основные понятия, законы, формулы по теме МАГНИТНОЕ ПОЛЕ и применить их на практике.

Слайд 5

Блок №1. Проверка домашнего задания . Критерии оценивания: Все выполнено верно, решал сам – 8б Все выполнено верно, воспользовался помощью, разобрался в решении – 7б Выполнил верно 7 заданий – 7б Выполнил верно 6 заданий — 6б Выполнил верно 5 заданий – 5б Выполнил верно 4 задания — 4б Выполнил верно 3 задания — 3б Выполнил верно 2 задания — 2б Выполнил верно 1 задание – 1б Не выполнил домашнее задание – 0б

Слайд 6

Блок заданий №2 Блиц-опрос на знание теории 1 .Что такое магнитное поле и каковы его свойства ? 2.Что такое магнитные линии? Каковы их особенности? 3. Что принято за направление вектора магнитной индукции? 4. Правило правой руки. 5. Правило левой руки.

Слайд 7

Оцените согласно критериям знания общих вопросов темы 5б — я знаю ответы на все вопросы; 4б — я знаю ответ на 4 вопроса; 3б – я знаю ответ на 3 вопроса; 2б – я знаю ответ на 2 вопроса; 1б – я знаю ответ на 1 вопрос; 0б – я не знаю ответ ни на один вопрос

Слайд 8

Блок №3 Вспомним формулы Величина Формула Сила Ампера Сила Лоренца Радиус окружности по которой движется заряженная частица в магнитном поле Период обращения заряженной частицы в магнитном поле Суммарная сила, действующая на заряд

Слайд 9

Блок №3 Вспомним формулы Величина Формула Сила Ампера F a =IB Δ lsin α Сила Лоренца F л =q υ Bsin α Радиус окружности по которой движется заряженная частица в магнитном поле Период обращения заряженной частицы в магнитном поле Суммарная сила, действующая на заряд F=F эл +F л

Слайд 10

Оцени знание формул. Критерии: 5б – все формулы записаны верно; 4б – записаны верно 4 формулы; 3б – записаны верно 3 формулы; 2б – записаны верно 2 формулы: 1б – записана верно 1 формула; 0б – нет верных ответов;

Слайд 11

Блок №4 Самостоятельная работа .

Слайд 12

Задача №1 Электрон, влетевший в зазор между полюсами электромагнита, имеет горизонтальную скорость и, перпендикулярную вектору индукции В магнитного поля (см. рис.).Куда направлена действующая на него сила Лоренца F ? Ответ:__________

Слайд 13

Задача №2 Электрическая цепь, состоящая из прямолинейных горизонтальных проводников и источника постоянного тока, находится в однородном магнитном поле, вектор индукции которого направлен горизонтально вправо (см. рисунок, вид сверху). Укажите, куда относительно рисунка направлена, вызванная этим полем сила Ампера, действующая на проводник 1-2. Ответ:____

Слайд 14

Задача №3 Протон р имеет скорость , направленную горизонтально вдоль прямого длинного проводника с током / (см. рисунок). Куда направлена действующая на протон сила Лоренца?

Слайд 15

Задача на соответствие №4 Частица массой m , несущая заряд q . движется в однородном магнитном поле с индукцией В по окружности радиуса R со скоростью v .Что произойдет с радиусом орбиты, периодом обращения и кинетической энергией частицы при уменьшении скорости движения? Для каждой величины определите соответствующий характер изменения: 1)увеличится 2)уменьшится 3)не изменится Запишите в таблицу выбранные цифры для каждой физической величины. Цифры в ответе могут повторяться.

Слайд 16

Задача на соответствие №5 Протон в однородном магнитном поле движется по окружности определенного радиуса. В это же поле с той же скоростью влетает а-частица. Радиус окружности, центростремительное ускорение и период обращения а-частицы по сравнению с протоном (заряд а- частицы в 2 раза больше заряда протона, а масса а-частицы в 4 раза больше его массы): 1)увеличится 2)уменьшится 3)не изменится Запишите в таблицу выбранные цифры для каждой физической величины. Цифры в ответе могут повторяться.

Слайд 17

Проверь себя: Верные ответы: 1 . От нас 2 . От нас 3 . Вверх 4 . 232 5 . 123 Критерии оценивания : №1- 1б №2- 1б №3- 1б №4,5 -3 верных ответа из 3-х — 2б 2 верных ответ из 3-х — 1б, 1верный ответ или нет верного ответа – 0 баллов

Слайд 18

Блок №5 Работа в группах Экспериментальное задание. Качественные задачи.

Слайд 19

Качественная задача №1 Мягкая пружина из нескольких крупных витков провода подвешена к потолку. Верхний конец пружины подключается к источнику тока через ключ К. а нижний — с помощью достаточно длинного мягкого провода (см. рис.). Как изменится длина пружины через достаточно большое время после замыкания ключа К? Ответ поясните, указав, какие физические явления и закономерности вы использовали для объяснения.

Слайд 20

Качественная задача №2 Рамку с постоянным током удерживают неподвижно в поле полосового магнита (см. рис.). Полярность подключения источника тока к выводам рамки покатана на рисунке. Как будет двигаться рамка на неподвижной оси МО, если рамку не удерживать? Ответ поясните, указав, какие физические закономерности вы использовали для объяснения. Считать, что рамка испытывает небольшое сопротивление движению со стороны воздуха

Слайд 21

Критерии оценивания задач (для тех, кто верно решил задачу) 3б — активно участвовал в работе группы, выдвигал гипотезу, обосновывал решение; 2б — участвовал в работе группы; 1б- наблюдал за процессом.

Слайд 22

Экспериментальное задание : Измерение поля постоянного магнита Цель: Продемонстрировать зависимость индукции магнитного поля от расстояния до постоянного магнита 22

Слайд 23

Критерии оценивания: 3б — активно участвовал в работе группы, выдвигал гипотезу, проводил эксперимент, формулировал выводы; 2б — участвовал в работе группы, помогал выдвигать, проводить, формулировать; 1б- наблюдал за процессом. 23

Слайд 24

Проект: Влияние искусственных и естественных электромагнитных колебаний на живые организмы. ( Прохоров К)

Слайд 26

Блок №6 Решение вычислительных задач

Слайд 27

Ответы: 72мН 5мТл 45 градусов 4 5А 480000 м/с 0,058м 16

Слайд 28

Резерв Решение расчетных задач на движение заряженной частицы в магнитном поле , по материалам ЕГЭ (повышенный уровень сложности) Проверка у доски. Электрон, разогнанный разностью потенциалов U , влетает в магнитное поле с индукцией В перпендикулярно линиям магнитной индукции. Определите радиус окружности R , которую опишет электрон.

Слайд 29

Рефлексия Заполнить карту рефлексии, ответив на вопросы и сдать учителю Чему научился сегодня на уроке, какой информацией овладел ? Кто, по вашему мнению, был самым сильным в группе ? Как ты оцениваешь свою работу на уроке ? На что необходимо обратить внимание в первую очередь ? Все ли было понятно на уроке ? Было ли интересно ? С каким настроением ты ушел с урока ?

Слайд 30

Оценки за урок 31б – 21б — «5» 20б – 16б – «4» 15б – 11б – «3» 10б и меньше – «2»

Слайд 31

Итоги урока . Оценки за урок Домашнее задание: 1) Р851. 2) Подготовить индивидуальные минипрезентации по теме « Влияние магнитного поля Земли на жизнедеятельность живых организмов »

Слайд 32

СПАСИБО ЗА УРОК!

Конспект урока по физике

Люгаева Алевтина Сергеевна

Преподаватель физики, астрономии, естествознания и общепрофессиональных дисциплин

ГАПОУ МО «Оленегорский горнопромышленный колледж»

г. Оленегорск Мурманской области

КОНСПЕКТ УРОКА

Тема урока: Решение задач: «Сила Ампера. Сила Лоренца».

Конспект урока рассчитан на 2 академических (учебных) часа.

Презентация к уроку: Сила Ампера. Сила Лоренца.
PPTX / 2.36 Мб

Дидактическая цель: создать условия для совершенствования знаний, умений, навыков.

Цель урока: совершенствовать навыки решения задач на применение силы Ампера и силы Лоренца; закрепить знание силы Ампера и силы Лоренца при решении задач; проверить уровень знаний, полученных при изучении данной темы.

Задачи урока:

Образовательные:

Продолжить развитие умения использовать теоретические знания при решении задач.

Продолжить формировать обобщенное умение решать задачи.

Сформировать умение решать задачи с использованием формул для расчета силы Ампера и силы Лоренца.

Проконтролировать степень усвоения знаний, умений и навыков по данной теме.

Развивающие:

Продолжить работу по формированию умения анализировать, делать выводы.

Воспитательные:

Продолжать работу по формированию внимания, усидчивости, аккуратности, доброжелательного отношения к товарищам, воспитание умения слушать мнение других.

— Совершенствовать навыки самостоятельной работы.

Личностные:

Способствовать саморазвитию и самообразованию учащихся на основе мотивации к обучению и познанию.

Формировать целостную картину мира.

Формировать осознанное, уважительное и доброжелательное отношение к другому человеку, его мнению.

Формировать умение контролировать процесс и результат деятельности (в частности, за счет рефлексии).

Метапредметные:

Использовать основные интеллектуальные операций: постановка задачи, формулирование гипотез, анализа и синтеза, сравнения, обобщения, систематизации, выявления причинно-следственных связей, поиска аналогов, формулирования выводов для решения задач.

Организовывать учебное сотрудничество и совместную деятельность с учителем и сверстниками.

Создавать, применять и преобразовывать знаково-символические средства для решения задач.

Самостоятельно планировать пути достижения целей

Предметные:

Владеть правилом левой руки, понятием Силы Ампера, Силы Лоренца.

Формировать умения решать задачи на тему «Сила Ампера. Сила Лоренца».

Использовать приобретенные знания в повседневной деятельности.

Формируемые УУД:

Личностные: Формирование мотивации изучения физики; стремление к самомовершенствованию; стремление к совершенствованию собственной речевой культуры в целом; развитие таких качеств как целеустремленность и инициативность.

Регулятивные: осуществление регулятивных действий самонаблюдения, самоконроля, самооценки в процессе коммуникативной и самостоятельной деятельности на уроке физики.

Познавательные: построение логических рассуждения, включающих анализ, синтез, установление причинно-следственныз связей, рефлексию.

Коммуникативные: планирование учебного сотрудничества, формулировка собственного мнения и позиции; способность аргументировать и координировать собственную позицию с позициями партнеров в сотрудничестве при выработке общего решения в совместной деятельности; умение задавать вопросы; адекватно использовать речевые средства для решения различных коммуникативных задач.

Тип урока: урок совершенствования знаний, умений и навыков.

Оборудование, материалы: проектор, компьютер, экран, доска, Касьянов В.А. Учебник Физика 11 класс, Степанова Г.Н.Сборник задач по физике, Рымкевич А.П.Задачник по физике.

Методы обучения: словесный, репродуктивный, проблемный (решение задач), частично-поисковый, эвристическая беседа.

Формы обучения: фронтальная, индивидуальная.

Методы контроля: письменный, устный.

Межпредметные связи: математика.

Методическое и дидактическое сопровождение урока:

— Презентация в Microsoft Power Point;

— Видеоролики на тему «Опыт Ампера», «Правило левой руки», «Электродвигатель».

— Компьютер, экран, мультимедиапроектор;

— Касьянов В.А. Учебник Физика 11 класс, Степанова Г.Н.Сборник задач по физике, Рымкевич А.П.Задачник по физике.;

— Раздаточный дидактический материал (конспект урока) для студентов.

СТРУКТУРА УРОКА

П/П

Этапы урока

Время (мин)

Задачи этапа

Деятельность

Метод

Организационная форма

преподавателя

обучающихся

1

Организационный этап

1

Проверить готовность к уроку

Приветствие. Проверка готовности обучающихся к работе

Проверяют готовность своего рабочего места

Информационный

Индивидуальная

2

Этап подготовки обучающихся к активному и сознательному усвоению знаний (целепологание и мотивация)

5

Подготовить к закреплению материала

В ходе беседы помогает сформулировать тему и цель занятия. Помогает сформулировать план занятия.

Отвечают на вопросы, определяют тему и цель занятия, и как может быть построено занятие

Словесный

Фронтальная

3

Актуализация знаний и умений

15

Повторить основные понятия и формулы

Предлагает ответить на вопросы

Отвечают на вопросы преподавателя. Вспоминают основные понятия и формулы по данной теме

Словесный (с использованием мультимедийной презентации)

Фронтальная

4

Этап решения типовых задач по теме: «Сила Ампера. Сила Лоренца».

32

Отработать умения решать задачи по данной теме

Организует работу обучающихся по решению задач и контролирует ход ее решения и результат

Совместное и самостоятельное решение задач по данной теме

Практический

Индивидуальная и фронтальная

5

Творческое применение и добывание знаний в новой ситуации

32

Отработать умения решать задачи по данной теме

Организует самостоятельную работу обучающихся по решению задач и контролирует

Самостоятельное решение задач по данной теме

Практический

Индивидуальная

6

Этап информации обучающихся о домашнем задании

2

Разъяснить методику выполнения домашнего задания

Задает домашнее задание

Определяют для себя уровень домашнего задания

Информационный

Индивидуальная

7Рефлексия, подведение итогов занятий3Осознать и оценить уровень сформированности навыков решения задачОрганизует обсуждение, оценивает степень выполнения решения задач, анализирует итоги урокаДают оценку результатов своей работы. Анализируют затруднения при решении задачАнализИндивидуальная

Ход урока:

1. Организационный момент. Приветствие. Проверка готовности обучающихся к работе

2. Этап подготовки обучающихся к активному и сознательному усвоению знаний (целепологание и мотивация) В ходе беседы помогает сформулировать тему и цель занятия. Помогает сформулировать план занятия.

3. Актуализация знаний:

Вокруг чего существует магнитное поле?

Чем характеризуется магнитное поле?

Что такое вектор магнитной индукции? Какой буквой обозначается? В каких единицах измеряется?

Сформулируйте правило буравчика?

Что такое линии магнитной индукции? Они всегда замкнуты или разомкнуты? Откуда выходят и куда входят линии магнитной индукции?

Что такое сила Ампера?

Сформулируйте закон Ампера? Формула? Как называется каждая буква в законе и в каких единицах измеряется?

Сформулируйте правило левой руки для проводника с током?

Что такое сила Лоренца? Формула? Как называется каждая буква в законе и в каких единицах измеряется?

Сформулируйте правило левой руки для заряженной частицы?

Какими формулами описывается движение частицы в магнитном поле по окружности? Из каких законов они выводятся?

4. Формирование умений.

Решение задач:

Рымкевич А.П.Задачник № 840, 841, 842

Касьянов В.А. Учебник. Физика 11 класс Стр.85 № 1,2

Вопросы к видеофильмам:

1. Опыт Ампера. https://www.youtube.com/watch?v=5HKE5TWF7nk

Описать установку действия магнитного поля на рамку с током?

Когда рамка отклоняется от положения равновесия?

Когда выталкивается рамка?

Причина отклонения рамки?

Как влияет сила тока в рамке на ее отклонение?

2.Правило левой руки. https://www.youtube.com/watch?v=ZNz5nhBKaGY

Сходство и различия правила левой руки, описанного в учебнике и описанного в видеофильме?

Когда сила Ампера не действует на проводник с током?

3. Электродвигатель. https://www.youtube.com/watch?v=rzOD8Mwlqm8

Какой закон применяется в электродвигателе?

5. Самостоятельная работа:

Степанова Г.Н.Сборник задач по физике

1 вариант № 1083, 1100, 1098, 1079, 1078 (1,3,5,7)

2 вариант № 1097, 1102, 1099, 1080, 1078 (2,4,6,8)

6. Этап информации обучающегося о домашнем задании.

Ваше домашнее задание: решить к следующему занятию контрольную работу по теме «Сила Ампера. Сила Лоренца».

7. Подведение итогов урока. Анализ результатов. Рефлексия учащихся.

Поднимите руки кто решил 5 задач? Кто 4? Кто 3? (5 задач – оценка 5, 4 задачи – оценка 4, 3 задачи – оценка 3).

Что вам более всего удалось, какие моменты были выполнены наиболее успешно?

Перечислите в порядке убывания основные трудности, которые вы испытывали во время самостоятельной работы. Как вы их преодолевали?

Какая сегодня была тема урока?

Какая сегодня была цель урока?

Заполните карточку:

Магнетизм — Физика — Теория, тесты, формулы и задачи

Оглавление:

 

Основные теоретические сведения

Сила Ампера

К оглавлению…

Заряженные тела способны создавать кроме электрического еще один вид поля. Если заряды движутся, то в пространстве вокруг них создается особый вид материи, называемый магнитным полем. Следовательно, электрический ток, представляющий собой упорядоченное движение зарядов, тоже создает магнитное поле. Как и электрическое поле, магнитное поле не ограничено в пространстве, распространяется очень быстро, но все же с конечной скоростью. Его можно обнаружить только по действию на движущиеся заряженные тела (и, как следствие, токи).

Для описания магнитного поля необходимо ввести силовую характеристику поля, аналогичную вектору напряженности E электрического поля. Такой характеристикой является вектор B магнитной индукции. В системе единиц СИ за единицу магнитной индукции принят 1 Тесла (Тл). Если в магнитное поле с индукцией B поместить проводник длиной l с током I, то на него будет действовать сила, называемая силой Ампера, которая вычисляется по формуле:

где: В – индукция магнитного поля, I – сила тока в проводнике, l – его длина. Сила Ампера направлена перпендикулярно вектору магнитной индукции и направлению тока, текущего по проводнику. 

Для определения направления силы Ампера обычно используют правило «Левой руки»: если расположить левую руку так, чтобы линии индукции входили в ладонь, а вытянутые пальцы были направлены вдоль тока, то отведенный большой палец укажет направление силы Ампера, действующей на проводник (см. рисунок).

Если угол α между направлениями вектора магнитной индукции и тока в проводнике отличен от 90°, то для определения направления силы Ампера надо взять составляющую магнитного поля, которая перпендикулярна направлению тока. Решать задачи этой темы нужно так же как и в динамике или статике, т.е. расписав силы по осям координат или складывая силы по правилам сложения векторов.

Момент сил, действующих на рамку с током

Пусть рамка с током находится в магнитном поле, причём плоскость рамки перпендикулярна полю. Силы Ампера будут сжимать рамку, а их равнодействующая будет равна нулю. Если поменять направление тока, то силы Ампера поменяют своё направление, и рамка будет не сжиматься, а растягиваться. Если линии магнитной индукции лежат в плоскости рамки, то возникает вращательный момент сил Ампера. Вращательный момент сил Ампера равен:

где: S — площадь рамки, α — угол между нормалью к рамке и вектором магнитной индукции (нормаль — вектор, перпендикулярный плоскости рамки), N – количество витков, B – индукция магнитного поля, I – сила тока в рамке.

 

Сила Лоренца

К оглавлению…

Сила Ампера, действующая на отрезок проводника длиной Δl с силой тока I, находящийся в магнитном поле B может быть выражена через силы, действующие на отдельные носители заряда. Эти силы называют силами Лоренца. Сила Лоренца, действующая на частицу с зарядом q в магнитном поле B, двигающуюся со скоростью v, вычисляется по следующей формуле:

Угол α в этом выражении равен углу между скоростью и вектором магнитной индукции. Направление силы Лоренца, действующей на положительно заряженную частицу, так же, как и направление силы Ампера, может быть найдено по правилу левой руки или по правилу буравчика (как и сила Ампера). Вектор магнитной индукции нужно мысленно воткнуть в ладонь левой руки, четыре сомкнутых пальца направить по скорости движения заряженной частицы, а отогнутый большой палец покажет направление силы Лоренца. Если частица имеет отрицательный заряд, то направление силы Лоренца, найденное по правилу левой руки, надо будет заменить на противоположное.

Сила Лоренца направлена перпендикулярно векторам скорости и индукции магнитного поля. При движении заряженной частицы в магнитном поле сила Лоренца работы не совершает. Поэтому модуль вектора скорости при движении частицы не изменяется. Если заряженная частица движется в однородном магнитном поле под действием силы Лоренца, а ее скорость лежит в плоскости, перпендикулярной вектору индукции магнитного поля, то частица будет двигаться по окружности, радиус которой можно вычислить по следующей формуле:

Сила Лоренца в этом случае играет роль центростремительной силы. Период обращения частицы в однородном магнитном поле равен:

Последнее выражение показывает, что для заряженных частиц заданной массы m период обращения (а значит и частота, и угловая скорость) не зависит от скорости (следовательно, и от кинетической энергии) и радиуса траектории R.

 

Теория о магнитном поле

К оглавлению…

Магнитное взаимодействие токов

Если по двум параллельным проводам идёт ток в одном направлении, то они притягиваются; если в противоположных направлениях, то отталкиваются. Закономерности этого явления были экспериментально установлены Ампером. Взаимодействие токов вызывается их магнитными полями: магнитное поле одного тока действует силой Ампера на другой ток и наоборот. Опыты показали, что модуль силы, действующей на отрезок длиной Δl каждого из проводников, прямо пропорционален силам тока I1 и I2 в проводниках, длине отрезка Δl и обратно пропорционален расстоянию R между ними:

где: μ0 – постоянная величина, которую называют магнитной постоянной. Введение магнитной постоянной в СИ упрощает запись ряда формул. Ее численное значение равно:

μ0 = 4π·10–7 H/A2 ≈ 1,26·10–6 H/A2.

Сравнивая приведенное только что выражение для силы взаимодействия двух проводников с током и выражение для силы Ампера нетрудно получить выражение для индукции магнитного поля создаваемого каждым из прямолинейных проводников с током на расстоянии R от него:

где: μ – магнитная проницаемость вещества (об этом чуть ниже). Если ток протекает по круговому витку, то в центре витка индукция магнитного поля определяется по формуле:

Силовыми линиями магнитного поля называют линии, по касательным к которым располагаются магнитные стрелки. Магнитной стрелкой называют длинный и тонкий магнит, его полюса точечны. Подвешенная на нити магнитная стрелка всегда поворачивается в одну сторону. При этом один её конец направлен в сторону севера, второй — на юг. Отсюда название полюсов: северный (N) и южный (S). Магниты всегда имеют два полюса: северный (обозначается синим цветом или буквой N) и южный (красным цветом или буквой S). Магниты взаимодействуют так же, как и заряды: одноименные полюса отталкиваются, а разноименные – притягиваются. Невозможно получить магнит с одним полюсом. Даже если магнит разломать, то у каждой части будет по два разных полюса.

Вектор магнитной индукции

Вектор магнитной индукции — векторная физическая величина, являющаяся характеристикой магнитного поля, численно равная силе, действующей на элемент тока в 1 А и длиной 1 м, если направление силовой линии перпендикулярно проводнику. Обозначается В, единица измерения — 1 Тесла. 1 Тл — очень большая величина, поэтому в реальных магнитных полях магнитную индукцию измеряют в мТл.

Вектор магнитной индукции направлен по касательной к силовым линиям, т.е. совпадает с направлением северного полюса магнитной стрелки, помещённой в данное магнитное поле. Направление вектора магнитной индукции не совпадает с направлением силы, действующей на проводник, поэтому силовые линии магнитного поля, строго говоря, силовыми не являются.

Силовая линия магнитного поля постоянных магнитов направлена по отношению к самим магнитам так, как показано на рисунке:

В случае магнитного поля электрического тока для определения направления силовых линий используют правило «Правой руки»: если взять проводник в правую руку так, чтобы большой палец был направлен по току, то четыре пальца, обхватывающие проводник, показывают направление силовых линий вокруг проводника:

В случае прямого тока линии магнитной индукции — окружности, плоскости которых перпендикулярны току. Вектора магнитной индукции направлены по касательной к окружности.

Соленоид — намотанный на цилиндрическую поверхность проводник, по которому течёт электрический ток I. Магнитное поле соленоида подобно полю прямого постоянного магнита. Внутри соленоида длиной l и количеством витков N создается однородное магнитное поле с индукцией (его направление также определяется правилом правой руки):

Линии магнитного поля имеют вид замкнутых линий — это общее свойство всех магнитных линий. Такое поле называют вихревым. В случае постоянных магнитов линии не оканчиваются на поверхности, а проникают внутрь магнита и замыкаются внутри. Это различие электрического и магнитного полей объясняется тем, что, в отличие от электрических, магнитных зарядов не существует.

Магнитные свойства вещества

Все вещества обладают магнитными свойствами. Магнитные свойства вещества характеризуются относительной магнитной проницаемостью μ, для которой верно следующее:

Данная формула выражает соответствие вектора магнитной индукции поля в вакууме и в данной среде. В отличие от электрического, при магнитном взаимодействии в среде можно наблюдать и усиление, и ослабление взаимодействия по сравнению с вакуумом, у которого магнитная проницаемость μ = 1. У диамагнетиков магнитная проницаемость μ немного меньше единицы. Примеры: вода, азот, серебро, медь, золото. Эти вещества несколько ослабляют магнитное поле. Парамагнетики — кислород, платина, магний — несколько усиливают поле, имея μ немного больше единицы. У ферромагнетиков — железо, никель, кобальт — μ >> 1. Например, у железа μ ≈ 25000.

 

Магнитный поток. Электромагнитная индукция

К оглавлению…

Явление электромагнитной индукции было открыто выдающимся английским физиком М.Фарадеем в 1831 году. Оно заключается в возникновении электрического тока в замкнутом проводящем контуре при изменении во времени магнитного потока, пронизывающего контур. Магнитным потоком Φ через площадь S контура называют величину:

где: B – модуль вектора магнитной индукции, α – угол между вектором магнитной индукции B и нормалью (перпендикуляром) к плоскости контура, S – площадь контура, N – количество витком в контуре. Единица магнитного потока в системе СИ называется Вебером (Вб).

Фарадей экспериментально установил, что при изменении магнитного потока в проводящем контуре возникает ЭДС индукции εинд, равная скорости изменения магнитного потока через поверхность, ограниченную контуром, взятой со знаком минус:

Изменение магнитного потока, пронизывающего замкнутый контур, может происходить по двум возможным причинам.

  1. Магнитный поток изменяется вследствие перемещения контура или его частей в постоянном во времени магнитном поле. Это случай, когда проводники, а вместе с ними и свободные носители заряда, движутся в магнитном поле. Возникновение ЭДС индукции объясняется действием силы Лоренца на свободные заряды в движущихся проводниках. Сила Лоренца играет в этом случае роль сторонней силы.
  2. Вторая причина изменения магнитного потока, пронизывающего контур, – изменение во времени магнитного поля при неподвижном контуре.

При решении задач важно сразу определить за счет чего меняется магнитный поток. Возможно три варианта:

  1. Меняется магнитное поле.
  2. Меняется площадь контура.
  3. Меняется ориентация рамки относительно поля.

При этом при решении задач обычно считают ЭДС по модулю. Обратим внимание также внимание на один частный случай, в котором происходит явление электромагнитной индукции. Итак, максимальное значение ЭДС индукции в контуре состоящем из N витков, площадью S, вращающемся с угловой скоростью ω в магнитном поле с индукцией В:

 

Движение проводника в магнитном поле

К оглавлению…

При движении проводника длиной l в магнитном поле B со скоростью v на его концах возникает разность потенциалов, вызванная действием силы Лоренца на свободные электроны в проводнике. Эту разность потенциалов (строго говоря, ЭДС) находят по формуле:

где: α — угол, который измеряется между направлением скорости и вектора магнитной индукции. В неподвижных частях контура ЭДС не возникает.

Если стержень длиной L вращается в магнитном поле В вокруг одного из своих концов с угловой скоростью ω, то на его концах возникнет разность потенциалов (ЭДС), которую можно рассчитать по формуле:

 

Индуктивность. Самоиндукция. Энергия магнитного поля

К оглавлению…

Самоиндукция является важным частным случаем электромагнитной индукции, когда изменяющийся магнитный поток, вызывающий ЭДС индукции, создается током в самом контуре. Если ток в рассматриваемом контуре по каким-то причинам изменяется, то изменяется и магнитное поле этого тока, а, следовательно, и собственный магнитный поток, пронизывающий контур. В контуре возникает ЭДС самоиндукции, которая согласно правилу Ленца препятствует изменению тока в контуре. Собственный магнитный поток Φ, пронизывающий контур или катушку с током, пропорционален силе тока I:

Коэффициент пропорциональности L в этой формуле называется коэффициентом самоиндукции или индуктивностью катушки. Единица индуктивности в СИ называется Генри (Гн).

Запомните: индуктивность контура не зависит ни от магнитного потока, ни от силы тока в нем, а определяется только формой и размерами контура, а также свойствами окружающей среды. Поэтому при изменении силы тока в контуре индуктивность остается неизменной. Индуктивность катушки можно рассчитать по формуле:

где: n — концентрация витков на единицу длины катушки:

ЭДС самоиндукции, возникающая в катушке с постоянным значением индуктивности, согласно формуле Фарадея равна:

Итак ЭДС самоиндукции прямо пропорциональна индуктивности катушки и скорости изменения силы тока в ней.

Магнитное поле обладает энергией. Подобно тому, как в заряженном конденсаторе имеется запас электрической энергии, в катушке, по виткам которой протекает ток, имеется запас магнитной энергии. Энергия Wм магнитного поля катушки с индуктивностью L, создаваемого током I, может быть рассчитана по одной из формул (они следуют друг из друга с учётом формулы Φ = LI):

Соотнеся формулу для энергии магнитного поля катушки с её геометрическими размерами можно получить формулу для объемной плотности энергии магнитного поля (или энергии единицы объёма):

 

Правило Ленца

К оглавлению…

Инерция – явление, происходящее и в механике (при разгоне автомобиля мы отклоняемся назад, противодействуя увеличению скорости, а при торможении отклоняемся вперёд, противодействуя уменьшению скорости), и в молекулярной физике (при нагревании жидкости увеличивается скорость испарения, самые быстрые молекулы покидают жидкость, уменьшая скорость нагревания) и так далее. В электромагнетизме инерция проявляется в противодействии изменению магнитного потока, пронизывающего контур. Если магнитный поток нарастает, то возникающий в контуре индукционный ток направлен так, чтобы препятствовать нарастанию магнитного потока, а если магнитный поток убывает, то возникающий в контуре индукционный ток направлен так, чтобы препятствовать убыванию магнитного потока.

Правило Ленца для определения направления индукционного тока: возникающий в контуре индукционный ток имеет такое направление, что создаваемое им магнитное поле препятствует изменению магнитного потока, которое вызывало этот ток.

Конспект урока физики в 11 классе Решение задач на применение силы Ампера и силы Лоренца


Название предмета: «Физика».
Класс: 11
УМК: Физика. 11 класс. /Г.Я.Мякишев, Б.Б.Буховцев, В.М.Чаругин; под.ред. Н.А. Парфентьевой. – М.: Просвещение, 2014.
Уровень обучения: базовый.
Тема: «Решение задач на применение силы Ампера и силы Лоренца»
Цель урока: формировать умения определять направление силы Ампера и силы Лоренца и вычислять их значение, развивать навыки логического мышления.
Задачи :Образовательные — проверка знания силы Ампера и силы Лоренца и правила левой руки; основных формул по теме “Магнитное поле” и единиц измерения электродинамических величин;
Воспитательные — развитие умения работать индивидуально у доски и в тетради; аккуратно вести записи, общаться с одноклассниками и педагогом.
Развивающие — развитие речи, мышления, развитие умений анализировать условие задачи, предсказывать результаты, делать выводы, стоить план решения задачи.
Ход урока.
1. Актуализация опорных знаний
Фронтальный опрос. (слайд 3, 4)
1) Как называется сила, с которой магнитное поле действует на заряженную частицу?
2) От чего зависит модуль силы Лоренца?
3) Как рассчитать модуль Fл?
4) Как определяется направление Fл?
5) Изменяется ли модуль скорости в магнитном поле? Ее направление?
6) Как будет двигаться частица, если υ║B?
7) Как будет двигаться частица, если υ┴B?
8) Какой будет траектория, если 0°
Проверка основных формул (вставить пропущенные физические величины) (слайд 5)
В= F/I…; Fл= eB…sinα; T=2π…/υ; Fa=B……sinα; F=mац=m…/r.
2. Физическая пауза (интересные факты) (слайд 6) (сообщения)
Магнитная термоизоляция высокотемпературной плазмы (слайд 7)
Траектория заряженной частицы как бы навивается на линии магнитной индукции. Это явление используется в технике для магнитной термоизоляции высокотемпературной плазмы, то есть полностью ионизированного газа при температуре порядка 106 K.
Вещество в таком состоянии получают в установках типа «Токамак» при изучении управляемых термоядерных реакций. Плазма не должна соприкасаться со стенками камеры. Термоизоляция достигается путем создания магнитного поля специальной конфигурации. В качестве примера на рис. 1.18.6 изображена траектория движения заряженной частицы в магнитной «бутылке» (или ловушке).
Радиационные пояса Земли. (слайд 8)
Вблизи магнитных полюсов Земли космические заряженные частицы движутся по спирали (с ускорением). Заряженная частица, движущаяся с ускорением, является источником электромагнитных волн — возникает т.н. синхротронное излучение.
Столкновение заряженных частиц с атомами и молекулами из верхних слоев атмосферы приводит к возникновению полярных сияний.
Вблизи магнитных полюсов Земли космические заряженные частицы движутся по спирали (с ускорением). Заряженная частица, движущаяся с ускорением, является источником электромагнитных волн — возникает т.н. синхротронное излучение.
Столкновение заряженных частиц с атомами и молекулами из верхних слоев атмосферы приводит к возникновению полярных сияний.
Движение заряженных частиц в магнитном поле Земли. (слайд 9)
Вблизи магнитных полюсов Земли космические заряженные частицы движутся по спирали (с ускорением). Заряженная частица, движущаяся с ускорением, является источником электромагнитных волн — возникает т.н. синхротронное излучение.
Столкновение заряженных частиц с атомами и молекулами из верхних слоев атмосферы приводит к возникновению полярных сияний.
3. Решение задач.
а)Качественные задачи (по материалам ЕГЭ)
1. (слайд 11) Ион Na+ влетает в магнитное поле со скоростью v перпендикулярно линиям магнитной индукции магнитного поля с индукцией В. Радиус орбиты иона можно рассчитать из выражения
mv/qB ;
qB/mv
mvB/q
mvq/B
2. (слайд 12) Электрон и протон влетают в однородное магнитное поле перпендикулярно вектору магнитной индукции со скоростями v и 2v. Отношение модуля силы, действующей на электрон со стороны магнитного поля, к модулю силы, действующей на протон, равно
1. 4:1
2. 2:1
3. 1:1
4. 1:2
3. (слайд 13) Угол между проводником с током и направлением вектора магнитной индукции внешнего однородного магнитного поля увеличивается от 300 до 900. Сила Ампера при этом
Возрастает в 2 раза.
Убывает в 2 раза.
Не изменяется.
Убывает до нуля
4.(слайд 14)Магнитное поле создано в точке А двумя параллельными длинными проводниками с токами I1 и I2, расположенными перпендикулярно плоскости чертежа. Векторы В1 и В2 в точке А направлены в плоскости чертежа следующим образом.
1. Вектор В1 – вниз, вектор В2 – вверх;
Вектор В1 – вверх, вектор В2 – вверх;
Вектор В1 – вниз, вектор В2 – вниз;
Вектор В1 – вверх, вектор В2 – вниз.
б)Решение расчетных задач (по материалам ЕГЭ) (слайд 15)
№1. Прямолинейный проводник длиной l = 50 см, по которому про ходит ток силой I = 0,5 А, перемещается в однородном магнитном поле, модуль индукции которого В = 0,2 Тл. Определите работу, совершаемую силой Ампера, при перемещении проводника на расстояние d = 40 см перпендикулярно линиям индукции.
Дано: l = 50 см, I = 0,5 А, В = 0,2 Тл, d = 40 см, А -?Решение:
А=FAd=BІlsin90°d=0,2∙0,5∙0,5∙1∙0,4=0,02 Дж==20 мДжОтвет: 20 мДж.
№2.В направлении, перпендикулярном линиям индукции в магнитное поле влетает электрон со скоростью 10 Мм/с, окружность какого радиуса описал электрон, если индукция поля 10мТл?
Дано: В=10мТл= 10-2 Тл;v=10Мм/с= 107м/с;е=1,6·10-19 Кл, me=9,1·10-31кг; R-?
Решение: Fл=maц; Fл=; aц=v2/R; mv2/R= evB; R=mv/eB;
R=9,1·10-31кг·107м/с/(1,6·10-19 Кл·10-2 Тл)=5,7·10-3м.
Ответ: 5,7·10-3м.
№ 3.Электрон движется в однородном магнитном поле с индукцией 4 мТл. Чему равен период обращения электрона?
Дано:
В= 4·10-3Тл; q= 1,6·10-19Кл; m= 9,1·10-31кг; Т – ?Решение. R= mV/q B; V=2πR/T; R=2πmR/TqB; T= 2π m/qB; T= 9·10-3c.
Ответ: 9·10-3c.
Изменится ли сила Лоренца, если в магнитное поле на тех же условиях влетит протон? Будет ли он двигаться по такой же окружности? С таким же периодом?
5.Рефлексия. Какой информацией ты овладел сегодня на уроке?
Кто, по вашему мнению, был самым сильным в группе?
Как ты оцениваешь свою работу на уроке?
На что необходимо обратить внимание в первую очередь?
Все ли было понятно на уроке?
Было ли интересно?
С каким настроением ты ушел с урока?
6.Подведение итогов урока
Домашнее задание: § 5, 6
Используемая литература:
Физика. 11 класс. /Г.Я.Мякишев, Б.Б.Буховцев, В.М.Чаругин; под.ред. Н.А. Парфентьевой. – М.: Просвещение, 2014.
ЕГЭ. Физика: типовые варианты: 30 вариантов/под ред. Демидовой М.Ю. – М.: Издательство «Национальное образование», 2017.

Сборник задач абитуриенту. МАГНЕТИЗМ. Сила Ампера. Индукция магнитного поля. Тема 22-1

          

МАГНЕТИЗМ. Сила Ампера. Индукция магнитного поля. Тема 22-1

22.1. На проводник длиной 0,5 м с током силой 20 A в однородном магнитном поле с индукцией 0,1 Тл действует сила 0,5 H. Какой угол (в градусах) составляет направление тока в проводнике с вектором магнитной индукции?

Ответ

22.2. Прямой проводник с током помещен в однородное магнитное поле перпендикулярно линиям индукции. Во сколько раз уменьшится сила, действующая на проводник со стороны магнитного поля, если его повернуть так, чтобы направление тока в проводнике составляло угол 30° с вектором индукции поля?

Ответ

22.3. Проводник с током помещен в однородное магнитное поле с индукцией B = 20 мТл. Определить силу, действующую на этот проводник, если его длина l = 0,1 м, сила тока I = 3,А, а угол между направлением тока и вектором B равен α = 45°.

Ответ

22.4. Проводник длиной 140 см согнули под прямым углом так, что одна из сторон угла равна 60 см, и поместили в однородное магнитное поле с индукцией 2 мТл обеими сторонами перпендикулярно линиям индукции. Какая сила (в мН) будет действовать на этот проводник, если по нему пропустить ток силой 10 A?

Ответ

22.5. Проводник длиной 110 см согнули под утлом 60° так, что одна из сторон угла равна 30 см, и поместили в однородное магнитное поле с индукцией 2 мТл обеими сторонами перпендикулярно линиям индукции. Какая сила (в мН) будет действовать на этот проводник, если по нему пропустить ток силой 10 A

Ответ

22.6. Определить направление силы взаимодействия тока с магнитным полем для каждого из случаев показанных на рисунке.

Ответ

22.7. Определить направление магнитного поля для каждого из случаев, показанных на рисунке. Проводник помещают перпендикулярно магнитному полю.

Ответ

22.8. По очень длинному вертикальному проводнику снизу вверх проходит ток силой 4 A. Определить положение точки вблизи проводника, в которой результирующий вектор магнитной индукции полей Земли и проводника имеет вертикальное направление. Горизонтальная составляющая индукции магнитного поля Земли 20 мкТл, провод расположен в северном полушарии.

Ответ

22.9. Два прямолинейных параллельных проводника с одинаковыми токами находятся на расстоянии 8 см друг от друга и притягиваются с силой 2,5 мН. Определить силу тока в проводниках, если их длина 250 см, а токи идут в одну сторону.

Ответ

СИЛА АМПЕРА. СИЛА ЛОРЕНЦА — Электромагнитное поле — ЭЛЕКТРОДИНАМИКИ — Физика 11 класс

ЭЛЕКТРОДИНАМИКИ

 

Глава 2 Электромагнитное поле

 

§ 14. СИЛА АМПЕРА. СИЛА ЛОРЕНЦА

 

Одним из проявлений магнитного поля является его силовое действие на движущиеся электрические заряды и проводники с током. В 1820 г. А. Ампер установил закон, определяющий силу, действующую на отрезок проводника с током в магнитном поле.

Так как создать обособленный элемент тока нельзя, то Ампер изучал поведение подвижных проволочных замкнутых контуров различной формы. Он установил, что на проводник с током, помещенный в однородное магнитное поле, индукция которого В, действует сила, пропорциональная длине отрезка проводника М, силе тока i, проходящего по проводнику, и индукции магнитного поля В:

это выражение называют законом Ампера, а силу — силой Ампера. Здесь а — угол между направлением тока в проводнике и направлением вектора Эта сила будет иметь максимальное значение при Если же проводник размещен вдоль линий магнитной индукции, то эта сила равна нулю.

Направление силы Ампера определяется с помощью правила левой руки, с которым вы ознакомились ранее (см. рис. 54).

Как мы уже выяснили, магнитное поле взаимодействует только с проводниками, через которые течет ток, и не влияет на проводники без тока. Это свидетельствует о том, что магнитное поле действует не на материал проводника, а на заряженные частицы (электроны или ионы), которые в нем перемещаются. В таком случае сила Ампера является результирующей всех сил, действующих на отдельные движущиеся заряженные частицы.

Определим силу, которая действует на заряженную частицу, которая движется в магнитном поле. Пусть на проводник длиной l, по которому течет ток силой i И который находится в магнитном поле с индукцией B, действует сила FA = BIlsina. Но сила тока определяется по формуле I = envS. Тогда FA = nevSBlsina, где произведение Sl — объем проводника, N = nSl — количество заряженных частиц, которые движутся в этом проводнике. Тогда силу, которая действует на одну движущуюся частицу, можно определить, поделив силу которая действует на все частицы, на общее количество подвижных заряженных частиц N;

Следовательно, сила, действующая на заряженную частицу, которая движется в магнитном поле, пропорциональна заряду частицы, скорости ее перемещения и индукции магнитного поля:

Эту формулу впервые получил голландский физик Г. Лоренц, и поэтому ее обычно называют формулой Лоренца, а силу, которая вычисляется по этой формуле, называют силой Лоренца.

Из формулы следует, что сила Лоренца зависит от угла между направлением движения заряженной частицы и направлением вектора индукции магнитного поля. Магнитное поле не действует на неподвижные заряженные частицы (Fл = 0 при v = 0) и на частицы, которые движутся вдоль линий индукции поля (sina = 0). Сила Лоренца будет максимальной, если частица влетает в магнитное поле перпендикулярно линиям его индукции. В этом случае FЛ = evB. Если поле однородно (В = const), то

 

 

Рис.60

 

Рис. 61

 

заряженная частица будет описывать окружность (рис. 60), поскольку движение под действием постоянной силы, направленного под прямым углом к скорости движения, согласно второму закону динамики, другим быть не может.

Направление силы Лоренца, как и силы Ампера, определяется по правилу левой руки: если левую руку разместить так, чтобы составляющая магнитной индукции В , перпендикулярная к скорости движения заряда, входила в ладонь, а выпрямленные четыре пальца были направлены по движению положительного заряда (против движения отрицательного), то отогнутый на 90° большой палец покажет направление силы Лоренца Fл. которая действует на заряд, помещенный в магнитное поле (рис. 61).

При этом нужно помнить, что это справедливо для положительно заряженных частиц. Если определяется направление силы Лоренца, действующей на электрон (или другую отрицательно заряженную частицу), то при применении правила нужно менять направление скорости движения на противоположный.

Действие силы Лоренца применяется во многих приборах и технических установках. Так, смещение электронного луча, который «рисует» изображение на экране кинескопа телевизора или дисплея компьютера, осуществляется магнитным полем специальных катушек, в которых проходит электрический ток, который изменяется по определенному закону. В научных исследованиях используются так называемые циклические ускорители заряженных частиц, в которых магнитное поле мощных электромагнитов удерживает заряженные частицы на круговых орбитах.

Это интересно знать

Исследования магнитного поля тока

1. «Правило пловца». В 1820 г. в Женеве на собрании натуралистов Араго увидел повторение опытов Эрстеда. И, вернувшись во Францию, решил ознакомить с ними своих соотечественников. Для этого он собрал нехитрую установку из вольто — вым столбом и продумал ход экспериментов.

Чтобы стрелка компаса легче вращалась, он заострил опорную иглу, включил ток, и магнитная стрелка начала отклоняться серебряным проводником с током от своего направления. Но что это? Араго протирает серебряный проводник и снимает с него прилеплены металлические опилки. Потом кладет проводник снова на стол, и опилки сразу же прилипают к нему… Араго выключает ток, и опилки осыпаются из серебряного проводника, включает — они облепляют проводник, словно бы серебро стало магнитом! Счастливое открытие, значение которого Араго понял сразу.

В тот момент, когда Араго закончил опыт, послышался стук в дверь. Араго выглянул в окно и увидел Ампера — академик Андре Мари Ампер — самый гениальный и найрозсіяніший из его друзей. Вид Ампера свидетельствовал однако, что ученый уже давно вышел из дома, бродя по улицам Парижа и размышляя над своими гениальными открытиями.

Ампер вошел в комнату, где проводил свои опыты Араго. «Я вижу, что Вы занимаетесь гальваническими опытами», — отметил Ампер. «Вы правы. Я воспроизвел опыт Эрстеда, и, как мне кажется, наткнулся на новое явление. Возможно оно заинтересует Вас», — ответил Араго. Он снова замыкает круг и приближает проводник до опилок, которые облепляют проводник, а когда выключает ток, то опилки высыпаются на ладони Ампера. Радостный ученый восклицает: «Прекрасно, это только лишний раз доказывает, что я прав. Заряды, которые находятся в покое, не взаимодействуют с магнитной стрелкой!» Ампер продолжает: «А как Вам кажется, будут ли взаимодействовать два проводника с током, как магниты?»

Не ожидая ответа, Ампер выходит из помещения Араго, у него в голове созрела новая гениальная мысль. Ученый шагает по набережной Сены, находясь в таком радостном и счастливом состоянии, которое бывает лишь тогда, когда становится понятным то, над чем долго и упорно работаешь и думаешь. Вдруг Ампер обращает внимание на мальчиков, которые в реке плавали по течению, весело перекликаясь, и здесь ученому приходит в голову гениальная мысль о простое правило, с помощью которого можно всегда определить отклонения магнитной стрелки электрическим током. Он решил назвать это правило «правилом пловца». Если человек будет плести по течению, направление которого совпадает с направлением тока, то пловец всегда будет видеть, что северный конец магнитной стрелки отклоняется под действием этого тока вправо. Браво Ампер! Ученый оглянулся: как хорошо было бы все это нарисовать здесь, поставить стрелки, определить направления. Вот и кусочек мела нашелся в кармане, а вот и черная доска. Прохожие парижане начали удивленно оборачиваться на летней (не очень опрятно одетого) джентльмена, который с большим увлечением разрисовывал мелом … заднюю стенку черной кареты. 18 июня 1820 г. на заседании Парижской академии наук А. Ампер начал свою знаменитую серию докладов по электромагнетизму.

2. «Правило свердлика».

На рис. 62 схематически проиллюстрировано «правило свердлика», открытое Дж. Мак — свеллом. Винт движется вниз по часовой стрелке (с точки зрения наблюдателя, смотрящего в направлении поступательного движения). Такое же направление имеет магнитный поток, что вызывает круговой ток.

3. «Правила правой и левой руки». До двух правил, открытых Ампером («правило пловца») и Максвеллом («правило свердлика») английский ученый, физик и электротехник Д. Флеминг добавил два других: «правила правой и левой руки» для определения

 

 

Рис. 62

 

 

Рис. 63

 

направления тока, магнитного поля и направления движения проводника под воздействием магнитного поля. На рис. 63 предоставлено схему, которая иллюстрирует эти два правила Флеминга.

На верхнем рисунке сила взаимодействия полей магнита и проводника с током приводит магнит в движение. Направление движения проводника определяется по «правилу левой руки» (двигатель), на нижнем рисунке направление электродвижущей силы, индуцированной в замкнутом контуре при его перемещении в магнитном поле, определяется по «правилу правой руки» (генератор).

Задачи и упражнения

Решаем вместе

1. Которые из заряженных космических частиц — с большей или меньшей скоростью движения, — пролетающие вблизи экваториальной плоскости Земли, ближайшее проникают к ее поверхности? Почему?

Решения

Поскольку радиус круга, который описывает заряженная частица в магнитном поле, r~ v, то ближе всего к Земле подлетят самые быстрые частицы.

2. Прямой провод длиной 10 см, по которому течет ток 0,5 А, находится в однородном магнитном поле перпендикулярно к силовым линиям. Определите индукцию магнитного поля, если оно действует на проволока силой 2,6 мН.

Решения

Сила, с которой однородное магнитное поле действует на прямой проводник, определяется по формуле Ампера

Отсюда определим

Подставив значения известных физических величин, получим В = 52 мТл.

3. Протон, проходя прискорюючу разность потенциалов 400 В, влетел в однородное магнитное поле с индукцией 0,2 Тл и начал двигаться по окружности. Определите радиус круга, по которому он двигался.

Решения

На заряженную частицу, которая влетела в магнитное поле действует сила Лоренца

По условию задачи протон движется по окружности, сила Лоренца является доцетровою силой Следовательно, откуда

Протон получил скорость, пройдя прискорюючу разницу потенциалов. По закону сохранения энергии работа, которую выполняет поле при перемещении протона, равна кинетической энергии, приобретенной протоном, то есть, А работа сил электрического поля при перемещении заряда определяется по формуле A = qU. Тогда

откуда

Следовательно, радиус круга, по которому двигался протон, определим по формуле

Подставив значения известных физических величин, получим R = 1,45 см.

11,5: Магнитная сила на проводнике с током

Движущиеся заряды испытывают силу в магнитном поле. Если эти движущиеся заряды находятся в проводе, то есть если по проводу проходит ток, на провод также должна действовать сила. Однако, прежде чем обсуждать силу, действующую на ток со стороны магнитного поля, мы сначала исследуем магнитное поле, создаваемое электрическим током. Здесь мы изучаем два отдельных эффекта, которые тесно взаимодействуют: провод с током создает магнитное поле, а магнитное поле оказывает силу на провод с током.

Магнитные поля, создаваемые электрическим током

Обсуждая исторические открытия в области магнетизма, мы упомянули открытие Эрстеда о том, что провод, по которому проходит электрический ток, вызывает отклонение расположенного рядом компаса. Было установлено, что электрические токи создают магнитные поля. (Эта связь между электричеством и магнетизмом более подробно обсуждается в Источниках магнитных полей.)

Стрелка компаса рядом с проволокой испытывает силу, которая выравнивает касательную иглы к окружности вокруг проволоки.Следовательно, токоведущий провод создает кольцевые петли магнитного поля. Чтобы определить направление магнитного поля, создаваемого проводом, мы используем второе правило правой руки. В RHR-2 ваш большой палец указывает в направлении тока, в то время как ваши пальцы охватывают провод, указывая в направлении создаваемого магнитного поля (рисунок \ (\ PageIndex {1} \)). Если магнитное поле попадало на вас или выходило за пределы страницы, мы обозначаем это точкой. Если магнитное поле попадало на страницу, мы обозначаем это знаком ×.

Эти символы получены с учетом векторной стрелки: стрелка, направленная на вас, с вашей точки зрения будет выглядеть как точка или кончик стрелки. Стрелка, направленная от вас, с вашей точки зрения будет выглядеть как крест или знак ×. Составной эскиз магнитных кругов показан на рисунке \ (\ PageIndex {1} \), где показано, что напряженность поля уменьшается по мере удаления от провода петлями, которые расположены дальше друг от друга.

Рисунок \ (\ PageIndex {1} \): (a) Когда провод находится в плоскости бумаги, поле перпендикулярно бумаге.Обратите внимание на символы, используемые для поля, указывающего внутрь (например, хвоста стрелки), и поля, указывающего наружу (например, кончика стрелки). (б) Длинный и прямой провод создает поле с силовыми линиями магнитного поля, образующими кольцевые петли.

Расчет магнитной силы

Электрический ток — это упорядоченное движение заряда. Следовательно, провод с током в магнитном поле должен испытывать силу, создаваемую этим полем. Чтобы исследовать эту силу, давайте рассмотрим бесконечно малое сечение провода, как показано на рисунке \ (\ PageIndex {3} \).Длина и площадь поперечного сечения секции составляют дл и A соответственно, поэтому ее объем равен \ (V = A \ cdot dl \). Проволока сформирована из материала, который содержит n носителей заряда в единице объема, поэтому количество носителей заряда в секции составляет \ (nA \ cdot dl \). Если носители заряда движутся со скоростью дрейфа \ (\ vec {v} _d \), ток I в проводе равен (от тока и сопротивления)

\ [I = neAv_d. \]

Магнитная сила на любом отдельном носителе заряда равна \ (e \ vec {v} _d \ times \ vec {B} \), поэтому общая магнитная сила \ (d \ vec {F} \) на \ (nA \ cdot dl \) носителей заряда в сечении провода

\ [d \ vec {F} = (nA \ cdot dl) e \ vec {v} _d \ times \ vec {B}.\]

Мы можем определить dl как вектор длиной dl , указывающий вдоль \ (\ vec {v} _d \), что позволяет нам переписать это уравнение как

\ [d \ vec {F} = neAv_dd \ vec {l} \ times \ vec {B}, \] или

\ [d \ vec {F} = Id \ vec {l} \ times \ vec {B}. \ label {11.12} \]

Это сила магнитного поля на отрезке провода. Обратите внимание, что на самом деле это результирующая сила, действующая со стороны поля на сами носители заряда. Направление этой силы задается RHR-1, где вы указываете пальцами в направлении тока и сгибаете их к полю.Затем ваш большой палец указывает в направлении силы.

Рисунок \ (\ PageIndex {2} \): бесконечно малое сечение токоведущего провода в магнитном поле.

Чтобы определить магнитную силу \ (\ vec {F} \) на проводе произвольной длины и формы, мы должны интегрировать уравнение \ ref {11.12} по всему проводу. Если сечение провода прямое и B однородное, дифференциалы уравнения становятся абсолютными величинами, что дает нам

\ [\ vec {F} = I \ vec {l} \ times \ vec {B}.\]

Это сила, действующая на прямой провод с током в однородном магнитном поле.

Пример \ (\ PageIndex {1} \): уравновешивание гравитационных и магнитных сил на проводе с током

Провод длиной 50 см и массой 10 г подвешен в горизонтальной плоскости с помощью пары гибких проводов (рисунок \ (\ PageIndex {3} \)). Затем на проволоку действует постоянное магнитное поле величиной 0,50 Тл, которое направлено, как показано. Каковы величина и направление тока в проводе, необходимые для снятия напряжения в опорных выводах?

Рисунок \ (\ PageIndex {3} \): (a) Проволока, подвешенная в магнитном поле.(б) Схема свободного тела для проволоки.

Стратегия

Из диаграммы свободного тела на рисунке, натяжения в опорных выводах стремятся к нулю, когда гравитационная и магнитная силы уравновешивают друг друга. Используя RHR-1, мы обнаруживаем, что магнитная сила направлена ​​вверх. Затем мы можем определить ток I , приравняв две силы.

Решение

Приравняйте две силы веса и магнитной силы к проводу:

\ [мг = IlB.2)} {(0,50 \, m) (0,50 \, T)} = 0,39 \, A. \]

Значение

Это сильное магнитное поле создает значительную силу на длине провода, чтобы противодействовать его весу.

Пример \ (\ PageIndex {2} \): расчет магнитной силы на токопроводящем проводе

По длинному жесткому проводу, проложенному вдоль оси y , проходит ток 5,0 А, протекающий в положительном направлении y . (а) Если постоянное магнитное поле величиной 0.30 Тл направлено вдоль положительной оси x , какова магнитная сила на единицу длины на проводе? (b) Если постоянное магнитное поле 0,30 Тл направлено на 30 градусов от оси + x к оси + y , какова магнитная сила на единицу длины на проводе?

Стратегия

Магнитная сила, действующая на провод с током в магнитном поле, определяется выражением \ (\ vec {F} = I \ vec {l} \ times \ vec {B} \). Для части а, поскольку ток и магнитное поле перпендикулярны в этой задаче, мы можем упростить формулу, чтобы дать нам величину и найти направление через RHR-1.Угол θ составляет 90 градусов, что означает \ (sin \, \ theta = 1. \). Кроме того, длину можно разделить на левую часть, чтобы найти силу на единицу длины. Для части b текущая длина, умноженная на длину, записывается в обозначении единичного вектора, а также магнитное поле. После взятия перекрестного произведения направленность очевидна по результирующему единичному вектору.

Решение

  1. Начнем с общей формулы магнитной силы на проводе. Мы ищем силу на единицу длины, поэтому мы делим ее на длину, чтобы вывести ее в левую часть.Мы также устанавливаем \ (sin \, \ theta \). Следовательно, решением будет \ [F = IlB \, sin \, \ theta \] \ [\ frac {F} {l} = (5.0 \, A) (0.30 \, T) \] \ [\ frac {F} {l} = 1,5 \, Н / м. \] Направленность: Укажите пальцами в положительном направлении y и согните пальцы в положительном направлении x . Ваш большой палец будет указывать в направлении \ (- \ vec {k} \). Следовательно, с учетом направленности решение будет \ [\ frac {\ vec {F}} {l} = -1,5 \ vec {k} \, Н / м. \]
  2. Текущее значение, умноженное на длину, и магнитное поле записываются в виде единичного вектора.o) \ hat {i} \] \ [\ vec {F} / l = -1.30 \ hat {k} \, Н / м. \]

Значение

Это большое магнитное поле создает значительную силу на небольшой длине провода. По мере того, как угол магнитного поля становится более близким к току в проводе, на него действует меньшая сила, как видно из сравнения частей a и b.

Упражнение \ (\ PageIndex {1} \)

Прямой гибкий медный провод погружают в магнитное поле, направленное внутрь страницы.(а) Если ток в проводе течет в направлении + x , в какую сторону будет изгибаться провод? (b) В какую сторону изгибается провод, если ток течет в направлении — x ?

Решение

а. наклоняется вверх; б. наклоняется вниз

Пример \ (\ PageIndex {3} \): сила на круглом проводе

Круговая токовая петля радиусом R , по которой проходит ток I , расположена в плоскости xy . Постоянное однородное магнитное поле прорезает петлю параллельно оси y (рисунок \ (\ PageIndex {4} \)).Найдите магнитную силу на верхней половине петли, нижней половине петли и общую силу на петле.

Рисунок \ (\ PageIndex {4} \): петля из провода, по которой течет ток в магнитном поле.

Стратегия

Магнитная сила на верхнем контуре должна быть записана в терминах дифференциальной силы, действующей на каждый сегмент контура. Если мы интегрируем по каждому дифференциальному элементу, мы решаем общую силу на этом участке петли. Сила, действующая на нижнюю петлю, определяется аналогичным образом, и общая сила складывается из этих двух сил.

Решение

Дифференциальное усилие на произвольном отрезке проволоки, расположенном на верхнем кольце, составляет:

\ [dF = I B \, sin \, \ theta \, dl, \], где \ (\ theta \) — угол между направлением магнитного поля (+ y ) и отрезком провода. Дифференциальный сегмент расположен на том же радиусе, поэтому, используя формулу длины дуги, мы имеем:

\ [dl = Rd \ theta \]

\ [dF = IBR \, sin \, \ theta \, d \ theta. \]

Чтобы найти силу на отрезке, мы интегрируем по верхней половине круга от 0 до \ (\ pi \).0 sin \, \ theta \, d \ theta = IBR (-cos 0 + cos \ pi) = -2 IBR. \]

Чистая сила — это сумма этих сил, равная нулю.

Значение

Полная сила на любом замкнутом контуре в однородном магнитном поле равна нулю. Несмотря на то, что каждая часть петли имеет силу, действующую на нее, результирующая сила, действующая на систему, равна нулю. (Обратите внимание, что на петле есть чистый крутящий момент, который мы рассмотрим в следующем разделе.)

Магнитная сила и сила Лоренца: вопросы и ответы

Этот набор вопросов и ответов с множественным выбором (MCQ) по электромагнитной теории посвящен теме «Магнитная сила и сила Лоренца».

1. Найдите электрическую силу, когда на заряд 2C действует электрическое поле величиной 6 единиц.
a) 6
b) 3
c) 12
d) 24
Посмотреть ответ

Ответ: c
Объяснение: Электрическая сила определяется как F = qE, где q = 2C и E = 6 единиц. Таким образом, мы получаем F = 2 x 6 = 12 единиц.

2. Найдите магнитную силу, если заряд 3,5C с магнитной индукцией 4 единицы имеет скорость 2 м / с.
a) 14
b) 28
c) 7
d) 32
Посмотреть ответ

Ответ: b
Пояснение: Магнитная сила определяется как F = q (v x B), где q = 3.5C, v = 2 м / с и B = 4 единицы. Таким образом, получаем F = 3,5 (2 х 4) = 28 единиц.

3. Найдите электрическое поле при скорости поля 12 м / с и плотности потока 8,75 единиц.
a) 510
b) 105
c) 150
d) 165
Посмотреть ответ

Ответ: b
Объяснение: Напряженность электрического поля является произведением скорости и плотности магнитного потока, т. Е. E = vx B = 12 х 8,75 = 105 единиц.

4. Найдите силу Лоренца для заряда 2.5C, имеющего электрическое поле 5 единиц и магнитное поле 7.25 единиц со скоростью 1,5 м / с.
a) 39,68
b) 68,39
c) 86,93
d) 93,68
Посмотреть ответ

Ответ: a
Объяснение: Сила Лоренца дается формулой F = qE + q (vx B), это сумма электрических и магнитная сила. При замене q = 2,5, E = 5, v = 1,5 и B = 7,25, F = 2,5 (5) + 2,5 (1,5 x 7,25) = 39,68 единиц.

5. Сила, действующая на проводник длиной 12 см, имеющий ток 8A и плотность потока 3,75 единиц под углом 300 °, составляет
a) 1,6
b) 2
c) 1.4
d) 1,8
Посмотреть ответ

Ответ: d
Объяснение: Сила, действующая на проводник, определяется как F = BIL sin θ, где B = 3,75, I = 8, L = 0,12 и θ = 300. Мы получаем F. = 3,75 x 8 x 0,12 sin 30 = 1,8 единиц.

6. Сила на единицу длины двух проводников, по которым проходят равные токи 5 А, разделенные на расстоянии 20 см в воздухе (порядок 10 -6 )
a) 25
b) 35
c) 40
d) 50
Посмотреть ответ

Ответ: a
Объяснение: Сила на единицу длины двух проводников определяется как
F = μ I1xI2 / 2πD, где I1 = I2 = 5 и D = 0.2. Таким образом, F = 4π x 10 -7 x 52 / 2π x 0,2 = 25 x 10 -6 единиц.

7. Когда токи движутся в одном направлении в двух проводниках, тогда сила будет
a) Притягивающая
b) Отталкивающая
c) Втягивающая
d) Противоположная
Посмотреть ответ

Ответ: a
Объяснение: Когда два проводника Если токи движутся в одном направлении, силы двух проводников будут двигаться навстречу друг другу или притягиваться.

8.Найдите плотность потока от проводника длиной 6 м, по которому проходит ток 3A (в порядке 10 -7 )
a) 1
b) 10
c) 100
d) 0,1
Посмотреть ответ

Ответ: a
Пояснение: Плотность потока равна B = мкГн, где H = I / 2πR. Положив I = 3 и R = 6, мы получим B = 4π x 10 -7 x 3 / 2π x 6 = 1 x 10 -7 единиц.

9. Найдите максимальное усилие проводника длиной 60 см, током 2,75 А и плотностью потока 9 единиц.
а) 14,85
б) 18.54
c) 84,25
d) 7,256
Посмотреть ответ

Ответ: a
Объяснение: Сила, действующая на проводник, определяется как F = BIL sin θ, где B = 3,75, I = 8, L = 0,12 и θ = 90 для максимальной силы. Получаем F
= BIL = 9 x 2,75 x 0,6 sin 90 = 14,85 единиц.

10. Магнитная сила воздействует на энергию поля. Состояние Истина / ложь.
a) Верно
b) Неверно
Посмотреть ответ

Ответ: a
Объяснение: Магнитная сила зависит от плотности потока материала, а плотность потока, в свою очередь, зависит от энергии материала.Можно показать, что F = q (v x B) и E = 0,5 x B 2 / μ. Ясно, что B и F связаны.

Sanfoundry Global Education & Learning Series — Электромагнитная теория.
Чтобы практиковать все области электромагнитной теории, представляет собой полный набор из 1000+ вопросов и ответов с несколькими вариантами ответов .

Примите участие в конкурсе сертификации Sanfoundry, чтобы получить бесплатную Почетную грамоту. Присоединяйтесь к нашим социальным сетям ниже и будьте в курсе последних конкурсов, видео, стажировок и вакансий!

Исходная сила Ампера и силы Био-Савара и Лоренца

  • (1)

    Дж.Д. Джексон: Classical Electrodynamics (Нью-Йорк, Нью-Йорк, 1975).

  • (2)

    Дж. К. Максвелл: Трактат об электричестве и магнетизме (Оксфорд, 1891 г. и Нью-Йорк, штат Нью-Йорк, 1954 г.).

  • (3)

    Р.А.Р. Трикер: Early Electrodynamics (Лондон, 1965).

  • (4)

    W. T. Scott: The Physics of Electricity and Magnetism (New York, N. Y., 1966).

  • (5)

    А. М. Ампер: Memoires de l’Academie Royale des Sciences (Париж, 1823 г.), издано в 1827 г.

  • (6)

    Р. К. Лайнесс: Contemp. Phys. , 3 , 453 (1961–1962).

    Google Scholar

  • (7)

    П. Уэсли: частное сообщение (1982).

  • (8)

    П. Грано: Nature (Лондон) , 295 , 311 (1982).

    ADS Статья Google Scholar

  • (9)

    В. К. Х. Панофски и М. Филлипс: Classifical Electricity and Magnetism (Reading, Mass., 1975).

  • (10)

    М. Алонсо и Э. J. Finn: Фундаментальная университетская физика , Vol. 2 (Ридинг, Массачусетс, 1978).

  • (11)

    Дж. М. Келлер: Am. J. Phys. , 10 , 302, (1942).

    ADS Статья Google Scholar

  • (12)

    Л. Пейдж и Н. И. Адамс: Am. J. Phys. , 13 , 141 (1945).

    ADS Статья Google Scholar

  • (13)

    А.M. Ampère: Memoires sur l’Electrodynamique , Vol. 1 (Париж, 1882 г.) с. 25.

    Google Scholar

  • (14)

    К. Геринг: Пер. Являюсь. Inst. Эл. Англ. , 42 , 311 (1923).

    Артикул Google Scholar

  • (15)

    Ф. Ф. Кливленд: Philos. Mag. , 21 , 416 (1936).

    Артикул Google Scholar

  • (16)

    W.Ф. Дантон: Nature (Лондон) , 140 , 245 (1937).

    ADS Статья Google Scholar

  • (17)

    И. А. Робертсон: Philos. Mag. , 36, , 32 (1945).

    Артикул Google Scholar

  • (18)

    P. T. Pappas: Nuovo Cimento B , 68, , 111 (1982) и ссылки в нем.

    ADS Статья Google Scholar

  • (19)

    Л.Д. Ландау и Э. М. Лифшиц: Классическая теория полей (Лондон, 1962).

  • (20)

    П. Лоррен и Д. Р. Корсон: Электромагнитные поля и волны (Сан-Франциско, Калифорния, 1970).

  • (21)

    П. Мун и Д. E. Spencer: J. Franklin Inst. , 257 , 203 (1954).

    Артикул Google Scholar

  • (22)

    П. Мун и Д. E. Spencer: J. Franklin Inst., 257 , 305 (1954).

    Артикул Google Scholar

  • (23)

    П. Мун и Д. E. Spencer: J. Franklin Inst. , 257 , 369 (1954).

    MathSciNet Статья Google Scholar

  • (24)

    Г. Б. Уокер и Г. Уокер: Nature (Лондон) , 263 , 401 (1976).

    ADS Статья Google Scholar

  • (25)

    Э.Kobel: Phys. Ред. , 36, , 1636 (1930).

    ADS Статья Google Scholar

  • (26)

    Дж. Д. Сетиан и Д. А. Хаммер: Phys. Rev. Lett. , 40, , 451 (1978).

    ADS Статья Google Scholar

  • (27)

    У. Г. Даффилд, Т. Х. Бернхэм и А. Х. Дэвис: Philos. Пер. R. Soc. Лондон сер. А , 220 А, 4, 109 (1926).

    Google Scholar

  • (28)

    H. M. P. Stoch: J. Phys. Д , 6, , 988 (1973).

    ADS Статья Google Scholar

  • (29)

    R. Tanberg: Phys. Ред. , 35, , 1080 (1930).

    ADS Статья Google Scholar

  • (30)

    Х. Аспден: IEEE Trans.Plasma Sci. ПС- 5 , № 3, сентябрь (1977).

  • (31)

    Х. Аспден: Physics Unified (Саутгемптон, 1980).

  • (32)

    Х. Аспден: Физика без Эйнштейна (Саутгемптон, 1969).

  • (33)

    H. Aspden: J. Franklin Inst. , 287 , 179 (1969).

    Артикул Google Scholar

  • класс силы Лоренца 12 глава

    Youtube видео — смотреть видео PDF ЗАМЕТКИ — ЗАГРУЗИТЬ PDF В этом мы рассмотрим темы — Физический класс 12, глава 4 (часть -1) Как магнитное поле создавалось. магнитное поле Величина магнитного поля, создаваемого 1 — круговой петлей 2 — круговой… Направленная вверх сила F величиной IlB должна быть уравновешена силой, обусловленной гравитацией: Mg = IlB B = Mg / Il = Магнитное поле равно 0.65 Т. 2. Единицей измерения магнитного поля в СИ является Вт · м-2 или Тл (телса). Давайте посмотрим на некоторые ключевые моменты главы 4: Глава состоит из множества прямых выводов, таких как закон оборота по амперу, сила… СИСТЕМА СВЯЗИ Алах Пандей Примечания к главам CBSE для всех глав 12-го класса физики доступны здесь. Магнитная сила Лоренца определяется выражением :; q — величина движущегося заряда. Совет штата Махараштра HSC Класс 12 Новая программа 2020-2021 Совет штата Махараштра H.S.C Класс 12 Новая программа 2020-2021 И Ма… Бухгалтерский учет и бухгалтерия Класс 11- Глава 2 -ЗНАЧЕНИЕ И ОСНОВЫ ДВОЙНОГО ВХОДА Что такое постоянные магниты? Контрольные работы за предыдущий год CBSE. Вопросы по физике класса 12, 2011 г. За пределами Дели. Контрольные работы за предыдущий год. CBSE. Класс 12, по физике, 2011 г., за пределами Дели. Вопрос 1.Коллектив имеет более 45 лет педагогического опыта. Электромагнитные волны Таблица содержания Токов смещенияУравнения Максвелла и сила Лоренца Источник электромагнитных волн Важные характеристики и свойства электромагнитных волн Интенсивность электромагнитных волн Импульс и радиационное давление электромагнитных волн Электромагнитный спектр Скачать рукописные заметки к главе 8 Физический класс 12-й Скачать Это ускорение теперь соответствует силе Лоренца: F ~ = @pi @t = e E + ~ v B (11.19) Цель этой главы — подробно показать, что сила Лоренца является результатом взаимодействующего уравнения Клейна-Гордона, которое описывает частицу без магнитного момента (спина) .Лоренц Форс. Курс быстрого пересмотра по физике 12 класса предоставляет вам всесторонний и исчерпывающий подход к концепциям физики класса 12 CBSE. Каким будет сила Лоренца для (а) электрона (б) протона 1 MCQ по физике для класса 12 Глава Мудрый с ответами Загрузка PDF-файла подготовлена ​​на основе последней схемы экзамена. В 12-м классе физики у вас будет 15 глав. 11 мин. Вопрос 5. Причину возникновения ЭДС можно объяснить на основе силы Лоренца. Узнайте больше об этом в Примечаниях к классу 12 «Движущиеся заряды и магнетизм».Ознакомьтесь с образцами вопросов из видео Lorentz Force Class 12 | EduRev для JEE, ответы и примеры объясняют значение главы… Магнетизм и материя Класс 12 Примечания включают определение силы Лоренца. Он действует на заряженную частицу из-за наличия магнитных полей и электрических полей. Решения NCERT … Как понять эту двигательную ЭДС, задействовав силу Лоренца, действующую на свободные носители заряда в проводнике? разность потенциалов и, следовательно, создает ЭДС ε = Blv….. (4) Поскольку эта ЭДС возникает из-за движения стержня, ее часто называют двигательной ЭДС. Видео на Youtube — смотреть видео PDF ЗАМЕТКИ — СКАЧАТЬ PDF В этом мы рассмотрим темы — Физический класс 12, глава 4 (часть -1) Как создавалось магнитное поле Возникшие эксперименты Закон Саварта Био Направление магнитного поля Величина магнитного поля, обусловленная 1 — — круговой цикл 2 — круговой… Наслаждайтесь любимыми видео и музыкой, загружайте оригинальный контент и делитесь всем этим с друзьями, семьей и всем миром на YouTube.Электрический ток создает магнитное поле, если два токоведущих проводника… 7.1k ДОЛЯ. Учащиеся должны быть хорошо знакомы со всеми концепциями главы 4 для решения дополнительных вопросов главы 4 по физике 12 класса. Максвелл был первым, кто рассчитал скорость распространения электромагнитных волн, которая была такой же, как скорость света, и пришел к выводу, что электромагнитные волны и видимый свет похожи. Класс 12, физика, электромагнитные волны и связь.Бесплатная загрузка PDF-файла CBSE Physics Вопросы с множественным выбором для класса 12 с ответами Глава 4 Движущиеся заряды и магнетизм. … в каком направлении будет сила Лоренца для (а) электрона (отрицательный заряд), (б) протона (положительный заряд). Studiestoday. Напишите его S.I.3. Его направление можно определить по правилу левой руки Флеминга. Учебники по физике отряда можно просмотреть, щелкнув имя отряда. Принцип формулировки циклотрона и энергии движущейся частицы. 4.2.2. Магнитное поле, сила Лоренца. 3. Давайте взглянем на некоторые ключевые моменты главы 4: Глава состоит из множества прямых выводов, таких как закон оборота по амперу, сила… Магнитное поле — это область, окружающая магнит.Сила, испытываемая движущимся зарядом в электрическом и магнитном поле, называется силой Лоренца. [1] Ответ: Электрический дипольный момент: Дипольный момент — это мера силы электрического диполя. Курс быстрого пересмотра по физике 12 класса предоставляет вам всесторонний и исчерпывающий подход к концепциям физики класса 12 CBSE. Программа CBSE по физике, класс 12. CBSE класс 12 Физические примечания к главе 4: Движущиеся заряды и магнетизм учебника физики NCERT класса 12 доступны в этой статье. Таким образом, в результате разделения зарядов на концах стержня возникает ЭДС.Сила Лоренца является основой всех расчетов орбиты в этой книге. Класс 12 Физика Движущиеся заряды Магнетизм Магнитное поле Магнитное поле — это эффект вокруг постоянного магнита или движущегося заряда, из-за которого ферромагнитные объекты, такие как некоторые металлы, притягиваются, а диамагнитные вещества отталкиваются при помещении в магнитное поле. КЛАСС — XII ФИЗИКА Глава -4: Движущиеся заряды и модуль магнетизма -1 (раздаточный материал) 1. Держите ручку и бумагу наготове для грубой работы, но держите книги подальше…. Эксперимент Эрстеда, движущийся заряд и магнитное поле, сила Лоренца, магнитная сила на проводнике с током, движение заряженной частицы в магнитном поле, циклотрон и т. Д. Первые 3 главы будут о переменном токе, атомах и коммуникации системы. ГЛАВА 2 Открытие электромагнетизма 2.1. ПОЛУПРОВОДНИКОВАЯ ЭЛЕКТРОНИКА: МАТЕРИАЛЫ, УСТРОЙСТВА И ПРОСТЫЕ ЦЕПИ Глава 15 и 8. Внешняя сила, необходимая для вращения стержня, противодействует силе Лоренца, действующей на стержень / внешняя сила действует в направлении, противоположном силе Лоренца.Команда DronStudy Academic состоит из преподавателей IITian, ​​известных Кота. EduRev похож на википедию только для образования и видео Lorentz Force Class 12 | Изображения и диаграммы EduRev даже лучше, чем Byjus! Опишите силу Лоренца. CBSE, класс 12, физика, мудрые примечания к главе 4: Движущиеся заряды и магнетизм учебника физики NCERT, класс 12, доступны в этой статье. Примечания к классу 12 магнетизма и материи включают определение силы Лоренца. Он действует на заряженную частицу из-за наличия магнитных полей и электрических полей.Если заряженная частица движется через магнитное поле, сила силы Лоренца будет отмечена как сила на отрицательном заряде противоположна силе на положительном заряде. Изучите концепции физики с помощью видеороликов и рассказов. Вопрос из раздела 12 класса МАГНИТНАЯ СИЛА, МОМЕНТ И МОМЕНТ Сила Лоренца определяется как 300+ ЛАЙКОВ. Класс 12. Темы и подтемы, затронутые в Примечания к классу 12 «Движущиеся заряды и магнетизм»: 4.1 Введение. Сила Лоренца Магнетизм и класс материи 12 Примечания включают определение силы Лоренца.Векторное произведение создает силу, обусловленную магнитом ХЕНДРИК АНТУН ЛОРЕНЦ (18 53 — 1928) Хендрик Антун Лоренц (1853 — 1928) голландский физик-теоретик, профессор в Лейдене. [1] Ответ: Электрический дипольный момент: Дипольный момент — это мера силы электрического диполя. Основы силы Лоренца. Магнитная сила Лоренца определяется выражением :; q — величина движущегося заряда. https://www.toppr.com/guides/physics/electromagnetism/lorentz-force B = Магнитные поля. Вопросы CBSE за предыдущий год Вопросы по физике 12 класса, Дели, 2013 год CBSE, за предыдущий год, вопросы по физике класса 12, за 2013 год, Delhi Set-I, Вопрос 1.Магнитная сила Лоренца, движущиеся заряды и магнетизм — получите заметки по темам, онлайн-тест, видеолекции, сомнения и решения для CBSE Class 12-science on TopperLearning. Согласно определению силы Лоренца, силы Лоренца — это силы, действующие на движущиеся заряды из-за электромагнитных полей. Эти заметки на основе NCERT соответствуют последней программе CBSE … • Сила Лоренца… Сила Лоренца дана. Уравнение силы Лоренца дается методом малого вывода. Важные вопросы для 5-й главы физики 12 класса повысят уверенность учащихся и позволят им решать все типы вопросов из 5-й главы 12-го класса … «Электрические заряды и поле» — это первая глава, которая касается электрических зарядов, полей и т. Д. и функциональные зоны…. Сила Лоренца на движущуюся частицу заряда, а также на неподвижную частицу. https://www.vedantu.com/revision-notes/cbse-class-12-physics-notes Получите общее выражение для радиуса круговой траектории. Ответы на задания даются в конце каждого задания с полными решениями. Книги. 4.2.3. Магнитная сила на проводнике с током. Вопросы с несколькими вариантами ответов с их ответами на chpater -4 Moving Charges and Magnetism для класса 12 NVERT. Изучите концепции движения зарядов и магнетизма в классе 12 по физике с помощью видеороликов и рассказов.Осознайте, что движущиеся заряды создают магнитное поле B (r) в дополнение к электрическому полю. Глава 20: Электромагнитная индукция. Сила Лоренца дается выражением. Он используется в электромагнетизме и также известен как электромагнитная сила. … Поскольку скорость: y электрона идет с запада на восток, выражение для магнитной силы Лоренца, то есть магнетизм силы Лоренца и примечания к классу материи 12, включают определение силы Лоренца. Загрузите CBSE Class 12 Physics Electric Charges And Fields Notes Set B в формате pdf, примечания к главе по физике, примечания к классу формулы интеллектуальных карт Примечания к редакции.Учащиеся должны быть хорошо знакомы со всеми концепциями главы 4 для решения дополнительных вопросов главы 4 по физике 12 класса. Проанализируйте рисунок и сопоставьте столбцы, приведенные ниже. NCERT Class 12 Книги по физике для слепых и слабовидящих студентов. Когда стержень вращается, свободные электроны в стержне перемещаются к внешнему концу из-за силы Лоренца и распределяются по кольцу. Полная сила, действующая на заряд q, движущийся с вектором скорости v в присутствии вектора B и вектора E магнитного и электрического полей, соответственно, называется силой Лоренца.Движущиеся заряды и магнетизм, глава 4, редакция PDF, класс 12, физика, глава 4, примечания к редакции, PDF, синее небо. Магнитное поле, создаваемое магнитами, оказывает на частицы силу, которая называется силой Лоренца. Лучшее приложение для студентов CBSE теперь содержит заметки по последней главе «Движущиеся сборы и магнетизм» для быстрой подготовки к экзаменам совета директоров CBSE и ежегодным экзаменам в школе. 7.1k ПРОСМОТРОВ. Хинди; Его возглавляет Нитин Агравал сэр — B.Tech IIT Madras / Автор 3 книг для IIT-JEE Physics / Преподавал в некоторых из лучших коучинговых институтов Индии / Имеет несколько изобретений (патентов) на свое имя в течение его корпоративных исследований.NCERT DC Панди Сунил Батра ХК Верма Прадип Безошибочный. Когда заряженная частица движется через область, в которой присутствуют как электрическое, так и магнитное поля, результирующая сила, испытываемая этой заряженной частицей, является суммой электростатической силы и магнитной силы и называется силой Лоренца. Всего в этом тесте будет 10 MCQ. Решения NCERT для физики класса 12 Глава 8 — Резюме главы. Концепция физики силы Лоренца Сила Лоренца — это сила, действующая на заряженную частицу за счет электрического и магнитного полей.Работа, совершаемая при перемещении заряда из P в Q, может быть выражена как W = QBvl. При определенном значении ЭДС поток электронов прекращается и достигается установившееся состояние. […] Сила на отрицательный заряд противоположна силе на положительном заряде. 4.2.1 Источники и поля. АТОМЫ Скачать PDF Глава 13. Движущиеся заряды и магнетизм, глава 4, редакция PDF, класс 12, физика, глава 4, примечания к редакции, PDF, blue sky, pcm. Примечания CBSE Class 12 Physics Chapter 4 Moving Charges and Magnetism в формате PDF доступны для бесплатной загрузки в мобильном приложении myCBSEguide.Решения NCERT для класса 12 по физике Глава 4 Движущиеся заряды и магнетизм. Конспект лекций. Полная сила или сила Лоренца, испытываемая зарядом q из-за электрического и магнитного полей, определяется как Следовательно, неподвижная заряженная частица не испытывает никакой силы в магнитном поле. 2) Определить двигательную ЭДС, индуцированную в прямом проводнике, движущемся в однородном магнитном поле с постоянной скоростью, на основе силы Лоренца. Контрольные работы за предыдущий год CBSE. Вопросы по физике класса 12, 2011 г. За пределами Дели. Контрольные работы за предыдущий год. CBSE. Класс 12, по физике, 2011 г., за пределами Дели. Вопрос 1.Какова формула силы Лоренца q = заряд. Загрузите CBSE Class 12 Physics Lorentz Force Notes в формате pdf, заметки к главам физики, заметки для классов формулы интеллектуальных карт Примечания к редакции Vidyakul — это онлайн-площадка для обучения и преподавания, которая предоставляет преподавателям платформу для создания онлайн-курсов и студентов, чтобы получить качественное образование по доступным ценам . 4.3. Движение в магнитном поле. Векторное произведение создает силу, обусловленную магнитным полем. ХЕНДРИК АНТУН ЛОРЕНЦ (18 53 — 19 28) Хендрик Антон Лоренц (1853 — 1928) голландский физик-теоретик, профессор из Лейдена.Сила, действующая из-за наличия магнитного поля, равна qv → × B →. Класс 12 по физике глубоко исследует различные концепции физики, в классе 12 в общей сложности 15 глав. 3. 4.2 Магнитная сила. (ii) Магнитная сила q [v × B] включает векторное произведение скорости и магнитного поля. … CBSE Class 12 Physics Lorentz Force Notes. 3) Первичная обмотка трансформатора имеет 40 витков и работает от 100 В и 100 Вт. Найдите число… Найдите наставника. PDF-файл «Важные вопросы», раздел 12, физика, глава 5, будет очень полезен для студентов, которые готовятся к экзаменам.Ответ: 1. Все видео здесь представлены в виде плейлиста. Кандидаты, стремящиеся попасть в класс 12 с хорошими баллами, могут проверить эту статью на наличие примечаний. Он задается как F = q (E * v * B) … Класс 12 Подробные ответы на вопросы учебников NCERT по главам представлены с целью помочь учащимся сравнить свои ответы с образцами ответов. Здесь V = 12 см с-1 = 12 x 10-2 мс-1 ∴ e = 0,50 x 12 x 10-2 x 15 x 10 2 = 9 x 10-3 В. Если q — это заряд электрона, то электроны в стержне будут испытывать магнитную силу Лоренца \ (- q [\ vec {v} + \ vec {B}] \) P.В. Поиск репетиторов Регистрация репетиторов Вход для репетиторов. Магнитное поле — это область вокруг магнита. Пошаговое решение вопросов цели -1 (MCQ-1) главы 34 Магнитное поле (концепция физики). Посетите официальный веб-сайт CISCE для получения подробной информации о классе плат ISC… В этом контексте магнитную силу можно описать как сила, возникающая из-за взаимодействующих магнитных полей. XII класс по физике; Блок 3-й; Магнитная сила Лоренца — YouTube Электромагнитная индукция. Важные вопросы для CBSE Class 12 Physics Electromagnetic Induction Laws.Например: Сталь, кобальт. Вопрос 2. Лекция 3: Получение закона силы Лоренца В этом разделе мы увидим, как электромагнетизм (ну, не весь электромагнетизм, но, по крайней мере, закон силы Лоренца) почти естественно следует из специальной теории относительности. A. Это векторная величина, […] Дайте силу Лоренца как сумму электрической силы и магнитной силы. Решения NCERT для класса 12 по физике Глава 4 Движущиеся заряды и магнетизм на английском языке Средний PDF-файл с важными вопросами для практики и ответами.1. [1] Ответ: Магниты с высокой удерживающей способностью и высокой коэрцитивной силой известны как постоянные магниты. Мы узнали, что движущиеся заряды окружают себя магнитным полем. Приведи один пример. В статье также представлена ​​формула непрерывного распределения заряда. Модульные видеоуроки для 12-го класса физики. Если магнитное поле параллельно положительной оси y, а заряженная частица движется вдоль положительной оси x, как показано на рисунке. Решения NCERT для математики 9 класса Глава 12; Решения NCERT для математики 9 класса Глава 13; Решения NCERT для математики 9-го класса Глава 14 Подробнее.Тест будет состоять только из вопросов с множественным выбором объективного типа, в которых учащиеся должны щелкнуть мышью по правильному выбору вариантов напротив соответствующего номера вопроса. Физика. Запишите его S.I. Центростремительная сила, необходимая для вращения, задается магнитной силой Лоренца. Следовательно, мы можем написать. Определить силу Лоренца: Закон силы Лоренца определяется как объединенная сила, испытываемая точечным зарядом из-за электрического и магнитного полей. Это набор дифференциальных уравнений в частных производных, которые составляют основу классической электродинамики, электрических цепей и классической оптики наряду с законом силы Лоренца.Суммарная сила, испытываемая зарядом, движущимся внутри электрического и магнитного полей, называется силой Лоренца. Поймите, что это векторное произведение и 0 для стационарных зарядов. Студенты могут решить NCERT Class 12 MCQ по физике движущихся зарядов и магнетизма в формате PDF с ответами, чтобы узнать свой […] Закон силы Лоренца. E = электрические поля. Из приведенного выше рисунка понятно, что магнитная сила перпендикулярна как магнитному полю, так и скорости заряда. Ниже приведены приложения силы Лоренца: Циклотроны и другие ускорители частиц используют силу Лоренца.В 1895 году Хендрик Лоренц вывел современную формулу силы Лоренца. Что такое формула силы Лоренца? Когда небольшой фрагмент распределения заряда делится на объем dV, получается следующая формула: С помощью правила правой руки становится легко найти направление магнитной части силы. 1. Замкнутая петля с током в форме равнобедренного прямоугольного треугольника ABC расположена в плоскости xy в однородном магнитном поле, действующем вдоль оси y, как показано на рисунке. Сила Лоренца на этом заряде определяется выражением: F = qvB.Магнитное поле Obj-1 HC Verma Solutions Vol-2 Class-12 Ch-34 Концепция физики для Class-12. … Сила, действующая на заряд в электрическом и магнитном поле (магнитная сила ЛОРЕНЦА) Правило Флемингса для левой руки за 1 минуту. Формула силы Лоренца. РЕЛЯТИВИСТСКИЙ ЭЛЕКТРОМАГНЕТИЗМ Таким образом, в этом случае величины связаны соотношением F = F0 cosh = 0I 2ˇr qv (8.24) Но это просто закон силы Лоренца F ~ = q ~ v B ~ (8.25) с B = jB ~ j, заданным по (8.2)! Направление магнитной силы перпендикулярно плоскости, содержащей вектор скорости и вектор магнитного поля.Анимация двигателя постоянного тока. CBSE, класс 12, физика, мудрые примечания к главе 4: Движущиеся заряды и магнетизм учебника физики NCERT, класс 12, доступны в этой статье. Блок. Мудрое и интеллектуальное применение ума может внести в это значительный вклад. Nulei Скачать PDF Глава 14. 46 ГЛАВА 8. В 1895 году Хендрик Лоренц вывел современную формулу силы Лоренца. 6.1 Импульс и Импульс. С более чем 35 лекциями, охватывающими каждую тему, представленную в программе CBSE Class 12 Physics Curriculum, можно легко пересмотреть весь курс в течение нескольких часов, и это тоже интересно.Лекционные заметки: Глава 12 Студенческие копии лекционных заметок [цветной pdf] … Лоренц Force. CBSE, класс 12, глава по физике, примечания по главе 4 «Движущиеся заряды и магнетизм» доступны здесь. Это дается выражением: F vector = q… Сила Лоренца определяется как комбинация магнитной и электрической сил, действующих на точечный заряд, вызванной электромагнитными полями. Примечания по физике для класса 12 Глава 4 Движущиеся заряды и магнетизм Эксперимент Эрстеда Магнитное поле создается вокруг любого проводника с током.С более чем 35 лекциями, охватывающими каждую тему, представленную в программе CBSE Class 12 Physics Curriculum, можно легко пересмотреть весь курс в течение нескольких часов, и это тоже интересно. Они могут осознавать движение в электрических и магнитных полях. Магнитная сила является следствием электромагнитной силы и возникает из-за движения зарядов. Важные вопросы для класса 12 по физике Глава 6 Электромагнитная индукция. Класс 12. Важные вопросы. Класс электромагнитной индукции 12. Важные вопросы. Очень короткий тип ответа.Какая сила движет частицу по круговой траектории и запишите математическое выражение. Магнитное поле и сила Магнитная сила была определена в Примечаниях к классу 12 главы 5 физики как притяжение и отталкивание, происходящие между электрически заряженными частицами. Это векторная величина, чья […] новая программа HSC Совета штата Махараштра, класс 12, 2020-2021 гг. ВХОД 2.1.1. Следовательно, конец P стержня станет положительным, а конец Q станет отрицательным. Рисунок: ЭДС движения от силы Лоренца V = El V = vBl Таким образом, сила Лоренца на свободных электронах отвечает за это. Мы пришли к выводу, что в лабораторной системе отсчета существует магнитная сила. Когда движущийся заряд помещен в однородное магнитное поле, на него действует сила, которая определяется выражением F = Bqvsinθ b. Изучите видео. Из этой части главы кандидаты узнают о концепции силы Лоренца и ее формуле…. Сила Лоренца, магнитная сила на … 2. Глава 12 Динамика релятивистских частиц и электромагнитных полей … u Закон силы Лоренца ddt = tu канонический импульс P j L uj = mujec A j P = pec A p = mu Гамильтониан HP u L = c P e A2 m2 c4 eu = c P e AP e A / c2 m2 c2 A = t1 t2 L dt = 1 2 L d L лоренц-инвариантен & A инвариантен L Momentum Intro задачи … Глава 12: Звук. Электронный луч, проецируемый вдоль оси + x, испытывает силу из-за магнитного поля вдоль оси + y. 2. ЭДС движения также можно объяснить на основе силы Лоренца, действующей на свободные носители заряда (свободные электроны) стержня XY.Магнитная сила Лоренца а. v = Скорость заряда. Блок. (Покажите, что магнитное поле направлено на доску, а скорость указывает вправо). Щелкните любое видео, чтобы воспроизвести полный список воспроизведения. Таким образом, чистая сила равна F → = qE → + qv → × B → F → = qE → + v → × B →, которая называется силой Лоренца. CBSE Class 12 Physics: Результаты обучения. 12.07 Радиус Бора и полная энергия электрона в модели атома водорода Бора 12.08 Уровни энергии 12.09 Константа Ридберга и линейные спектры атома водорода … 8.04 Уравнения Максвелла и текстовое решение силы Лоренца.Сила, действующая из-за наличия электрического поля, qE →. 2. Определите электрический дипольный момент. Учитесь с 6 по 12 класс и конкурсный экзамен. Магнитная сила Лоренца. Рабочие листы и лабораторные работы класса. Две полезные вспомогательные функции величин поля, Магнитные поля, как и гравитационные поля, нельзя увидеть или потрогать. NCERT Solutions for Class 12 Physics Chapter 4 в формате PDF для бесплатного скачивания на академической сессии 2021-22 гг. Он Все важные вопросы собраны в одном месте, чтобы помочь студенту пересмотреть концепции.Обзор. (ii) Магнитная сила q [v × B] включает векторное произведение скорости и магнитного поля. Закон Лоренца (раздел 3.2) описывает электромагнитные силы, действующие на частицу, как функцию полей и свойств пробной частицы (заряда, положения и скорости). Это редкий талант легко производить максимум из чего-либо. Объяснять. ДВОЙНАЯ ПРИРОДА ИЗЛУЧЕНИЯ И МАТЕРИИ Скачать PDF Глава 12. Скачать CBSE Class 12 Заметки о силе Лоренца в формате pdf, примечания к главе по физике, примечания к классу формулы интеллектуальных карт Примечания к редакции Определите электрический дипольный момент.ЭДС движения также можно объяснить на основе силы Лоренца, действующей на свободные носители заряда (свободные электроны) стержня XY. … Сила Лоренца, магнитная сила на … Принцип наименьшего действия. Глава 1: Электрические заряды и поле Примечания, сделанные вручную Электрический заряд, основные свойства электрического заряда и фрикционное электричествоЭлектрическая и электростатическая сила Закон Кулона и закон Кулонов в векторной формеПринцип суперпозицииЭлектрическое поле и расчет электрического поляЛинии электрического поляЭлектрический потокЭлектрический диполь и дипольный моментЗакон Гаусса и его применение Важно … Направление магнитной силы перпендикулярно плоскости, содержащей вектор скорости и вектор магнитного поля.Магнитное поле и сила Магнитная сила была определена в Примечаниях к классу 12 главы 5 физики как притяжение и отталкивание, происходящие между электрически заряженными частицами. Для начала предположим, что единственное направление этого магнитного поля может быть получено с помощью правила плавания Ампера. Когда вы объединяете это с электрической силой, вы получаете: самый интересный результат силы Лоренца или магнитной силы заключается в том, что она заставляет вещи двигаться по кругу, а магнитная сила не может делать никакой работы.В этих главах вы узнаете о переменном напряжении, трансформаторах, колебаниях LC, атомной модели Бора, электромагнитных волнах, амплитудной модуляции, основных системах связи и многом другом.

    Обсуждение типов засух в Южной Африке, Приложение Marquee Sports Network, Секретный чат Viber 2021, Недельное меню ресторана City Tavern, Лагеря вьетнамских беженцев в Таиланде,

    Устаревшая формула? · Histoire de l’électricité et du magnétisme (сайт Ampère)

    Accueil> Исторический парк… de la boussole à la Fée électricité> Des lois pour le courant: Ampère, Ohm et quelques autres …> Закон силы Ампера: устаревшая формула?

    Французский

    Кристин Блондель и Бертран Вольф
    Перевод Эндрю Бутрика

    Нередко в науке прошлого можно найти работы, которые никогда не считались недействительными, но исчезли из современной науки. Они могли быть отменены последующей работой, признанной более эффективной, или были оставлены, потому что считались малоинтересными, или были дискредитированы по ненаучным причинам, или просто были забыты.Их статус остался неопределенным. Однако в любой момент ученые могут взяться за столь невостребованную работу по разным причинам. Так обстоит дело с силой Ампера.

    Электродинамическая теория Ампера основана на существовании силы между двумя элементами тока, и математическое выражение этой силы подразумевает отталкивание между двумя выровненными элементами. Однако это взаимодействие между коллинеарными элементами исчезло с появлением элементарной силы Грассмана (1845), силы, которая теперь составляет неотъемлемую часть современной теории электромагнетизма.

    Чтобы выбрать между выражениями электродинамической силы Ампера и Грассмана, можно поискать эксперименты, основанные на существовании или отсутствии отталкивания между коллинеарными элементами электрической цепи. Тем не менее, прямая проверка формулы Ампера и ее применения к частному случаю коллинеарных элементов невозможна. Действительно, нельзя изолировать токовые элементы в электрической цепи. Однако Ампер думал, что должен иметь экспериментальное доказательство отталкивания между коллинеарными элементами.Мы обсудим различные эксперименты, предложенные в конце двадцатого века в продолжение эксперимента Ампера и, вероятно, продемонстрирующие существование этой силы. Интерпретация этих экспериментов до сих пор остается спорной.

    С теоретической точки зрения совместимость теории Максвелла-Лоренца и силы Ампера также является предметом споров.

    Наконец, для некоторых защита силы Ампера является частью более крупного проекта, который состоит из разработки — вслед за Вебером и Нойманом — «ньютоновской» альтернативы теории Максвелла, использующей только взаимодействия на расстоянии между материальными элементами без обращения к электромагнитному полю.Этот проект может показаться безрассудным и основанным на непрочном основании, в то время как теория Максвелла, связанная с теорией относительности Эйнштейна, составляет основную опору современной физики.

    Мы, тем не менее, заключаем, следуя самому Максвеллу, что желательно, чтобы сама жизнь науки оставила открытыми ряд путей: «» «Хорошо иметь два взгляда на предмет и признать, что есть два взгляда на это ». ( The Scientific Papers of James Clerk Maxwell , t.1, стр. 208)

    1873: «Формула, из которой могут быть выведены все явления …»

    По словам самого Максвелла, «теория Ампера» сводится к формуле, из которой могут быть выведены все явления, и которая всегда должна оставаться основной формулой электродинамики ». ( Трактат об электричестве и магнетизме , 1873, т. 2, с. 175).

    Эта фундаментальная формула Ампера выражает силу, оказываемую одним бесконечно малым идентификатором элемента тока на другой бесконечно малый элемент тока i’ds ‘, расположенный на расстоянии r друг от друга и относительное положение которых определяется тремя углами α, β и γ [ См. Страницу В поисках закона электродинамики Ньютона ].

    Элементарная сила Ампера между id и i’ds ‘:
    i i’ ds ds ‘(sinα sinβ cosγ — ½ cosα cosβ) / r 2

    Рис. 1. Текущий элемент ds, расположенный в точке A, расположен в плоскости P; элемент ds ‘, расположенный в точке B, находится в плоскости Q.

    По этой формуле Ампер мог вычислить путем двойного интегрирования все взаимодействия между реальными цепями.Применяя этот метод к небольшим круговым токам, которые, как он предполагал, существуют внутри магнитов, он получил законы взаимодействия между двумя магнитами или между магнитом и током. Опираясь на эту формулу, можно, как писал Максвелл, «вывести все явления».

    1958: формула, которая «больше не служит никакой цели»

    В 1958 году Эдмон Бауэр, который сотрудничал с Полем Ланжевеном над его теорией магнетизма, переработав фундаментальные идеи Ампера, переиздал великий синтез Ампера, Математическая теория электродинамических явлений, однозначно выведенных из эксперимента. В своем предисловии, которое в другом месте весьма хвалебно, Бауэр написал: «« Знаменитая формула Ампера больше не служит никакой цели »». Действительно, после преобладания во Франции до 1890-х годов теория Ампера постепенно была отброшена. В начале двадцатого века физик Анри Буасс в своем трактате об электричестве мог утверждать:

    «Формула Ампера больше не представляет никакого, кроме исторического интереса. Это пустая трата времени — обсуждать гипотезы, на которых основывался Ампер, чтобы установить это, или последствия, которые он определяет для действий между двумя элементами в определенных положениях.”

    Если и сегодня в некоторых текстах по физике можно найти «формулу Ампера», то в большинстве случаев это не оригинальная формула Ампера, а формула, предложенная в 1845 году немецким математиком Германом Грассманом. Этот пионер векторного анализа определил «геометрическое произведение», из которого позже можно было бы получить обычное векторное произведение. Благодаря использованию этой новой математической концепции, его сила, казалось, предлагала большую простоту, чем у Ампера. Но, в отличие от последнего, он не удовлетворял принципу действия и противодействия.

    Элементарная сила Грассмана (приложенная элементом к элементу ), в современных символах:

    Интенсивность силы: i i ‘ds ds’ sinα cosβ (β — угол между плоскостью P).

    Эта сила, выражение которой не приписывает симметричную роль двум элементам и всегда перпендикулярна элементу.

    1980-е: формула Ампера, новый научный вызов?

    С 1980-х годов определенное число физиков — при поддержке теоретических работ, а также экспериментальных результатов — считали, что нужно вернуть из забвения первоначальную формулу Ампера, которая теперь оказывается, спустя почти два столетия после ее «изобретения», предметом изучения. научная полемика.

    Некоторые даже утверждают, что некоторые эксперименты можно объяснить только формулой Ампера.

    Возвращаемся ли мы к архаичному прошлому, которое устарело электромагнетизмом Максвелла-Лоренца? Формула Ампера просто вышла из употребления, перестала быть оспоренной, опровергнута более плодотворной, эстетичной и простой в применении теорией, или она несовместима с современным электромагнетизмом, объясняющим множество явлений?

    Неопределенность суждения Максвелла

    Хотя Максвелл считал формулу Ампера «фундаментальной», он, тем не менее, добавил:

    «Следовательно, нельзя сказать, что утверждение о взаимном действии двух элементов схемы основано на чисто экспериментальных основаниях.»( Трактат … , т. 2, стр. 163)

    Действительно, поскольку электрические токи образуют замкнутые цепи, невозможно физически изолировать действие токового элемента (мы не рассматриваем здесь разомкнутые цепи в качестве радиоантенны, очага высокочастотных электрических колебаний). Хотя многие экспериментальные приемы позволяют сделать небольшую часть схемы (C ‘) подвижной, эта часть подвержена действию всей схемы (C), но невозможно изолировать действие одной из схем (C). элементы.

    Бесконечное количество элементарных формул, включая формулы Ампера и Грассмана, может привести к той же силе, действующей на элемент i’ds ‘путем интегрирования по всей цепи (C).

    Почему же тогда тот же Максвелл описывает формулу Ампера как «фундаментальную формулу электродинамики»? На самом деле это единственный метод, соблюдающий ньютоновский принцип действия и противодействия. Обычно сила Грассмана не переносится по прямой, соединяющей два элемента. Кроме того, сила Грассмана, прилагаемая элементом , к элементу, не имеет такой же интенсивности, как сила, действующая на элемент , , полученная путем изменения ролей двух элементов.

    Как сказал физик Р.А.Р. Трикер подчеркнул, что сегодня мы рассматриваем принцип Ньютона как применимый только к конечным силам, имеющим физическую реальность, следовательно, к силам между контурами:

    «Читатель вполне может придерживаться той точки зрения, что вопросы, которые даже в принципе не могут быть подвергнуты проверке опытом [например, правильная элементная формула], не относятся к науке, но, несмотря на это, Ампер подвергался значительная критика. Достаточно любопытно, что единственная критика, которой он был бы уязвим, заключалась в том, что он тоже, вместе с большинством его критиков, думал, что он пришел к закону силы, который нынешние элементы на самом деле оказывают друг на друга… »(R.A.R. Tricker, Early Electrodynamics, 1965, p. 99)

    Формула Ампера опровергнута уравнениями Максвелла и «силой Лапласа»

    Интегрируя силу Грассмана по контуру (C), можно получить силу, действующую этим контуром (C) на элемент контура (C ‘):

    Результат суммирования по (C) впоследствии был идентифицирован как магнитное поле, созданное контуром (C). Таким образом, сила Грассмана принимает простую векторную форму .Эта сила перпендикулярна и к, и ее интенсивность равна i’ds’B sinε, где ε — угол между и . По крайней мере, во Франции этот закон известен как «сила Лапласа».

    Сила Лапласа:

    Ампер установил аналогичный закон [см. Стр. В поисках … ]. Действительно, он показал, что сила, прилагаемая замкнутым контуром к элементу ds ‘, перпендикулярна этому элементу и прямой линии, которую он назвал директрисой .Он выразил интенсивность этой силы как:
    ½ D i i ‘ds’ sinε.
    В множителе ½ Di, связанном с директрисой , можно распознать характеристики магнитного поля B. Но Ампер увидел в этом законе только математическое следствие действия на расстоянии между элементами тока.

    Триумф теории Максвелла, основанной на непрерывном распространении электромагнитных воздействий, объясняет отказ от формулы Ампера в пользу формулы Грассмана, которая идеально вписывается в рамки Максвелла, потому что, как мы видели, легко связать ее с полем .Таким образом, дань уважения Максвеллу исторической формуле Ампера несколько парадоксальна.

    Силы отталкивания между коллинеарными элементами?

    Вопрос остается открытым. Можно ли, вопреки утверждениям Максвелла, выбрать между элементарными формулами Ампера и Грассмана экспериментальным путем? И если эксперимент решит в пользу силы Ампера, бросит ли это вызов сооружению, возведенному Максвеллом, которое само по себе неразрывно связано с теорией относительности?

    Формула

    Ампера предполагает, в отличие от формулы Грассмана, существование силы отталкивания между двумя коллинеарными элементами тока.Действительно, если два элемента коллинеарны, углы α и β (рис. 1) равны нулю, и сила между ds и ds ‘становится равной:
    — ½ i i’ ds ds ‘/ r 2 .
    Сила отрицательная, это отталкивание. Напротив, формула Грассмана, dF = i i ‘ds ds’ sinα cosθ, подразумевает, что сила между коллинеарными ds и ds ‘равна нулю.
    Согласно Ампера, в металлической проволоке, по которой проходит ток, должна существовать сила отталкивания между двумя последовательными элементами. Итак, можем ли мы выбрать между двумя формулами, изучая действие, осуществляемое между двумя частями одной и той же цепи?

    «Эксперимент с шпилькой» Ампера, повторенный в 1980-х годах

    В 1822 году Ампер решил, что доказывает свою формулу своим экспериментом с плавающим проводником, который иногда называют «экспериментом с шпилькой».[См. Видео L’expérience du conducteur flottant на странице В поисках … ].

    Рис. 2. Медный провод nqr, в форме шпильки , с изоляцией , за исключением оголенных концов n и r , плавает на поверхности ртути, содержащейся в двух участках кругового круга. контейнер. Когда через цепь протекает сильный ток, шпилька отходит от неподвижных контактов s и m независимо от направления тока.

    В 1980-х годах несколько исследователей пересмотрели этот эксперимент.

    — Питер Грано из Массачусетского технологического института наблюдал в 1981 году при токах в несколько сотен ампер сильную турбулентность в ртути, когда движение шпильки было заблокировано. Эта турбулентность, казалось, проявляла отталкивание части ртути, несущей ток

    .

    — Панос Т. Паппас в 1983 году заменил шпильку Ампера проволокой, сформированной в виде прямоугольника без одной из более коротких сторон и подвешенного в виде маятника (рис.3). Автор определил величину движения, сообщаемого маятнику, и приписал это движение силе отталкивания Ампера. Этот эксперимент был выполнен с большей точностью Питером и Нилом Грано (1986) с использованием разряда серии конденсаторов.

    Рис. 3. Принцип импульсного маятника Паппаса.
    Концы подвижного проводника находятся в электрическом контакте с неподвижными выводами батареи через маленькие стаканчики из ртути B и E. При включении тока маятник испытывает отталкивание, и электрический контакт разрывается.

    — Все еще в соответствии с экспериментом Ампера, другие физики пытались «взвесить» возможную силу отталкивания. Так, Питер Грано в 1986 году, а затем Реми Сомон в 1992 году подвесил такой же подвижный кондуктор в форме неполного прямоугольника вертикально под чашей точных весов. Затем можно было измерить вертикальную силу отталкивания, которую он испытывал от неподвижной части цепи.

    Противоречивые толкования

    Фактически, различные вариации эксперимента Ампера интерпретируются в контексте современной физики с использованием силы Лапласа и, следовательно, элементарной силы Грассмана.Действительно, поперечная часть неполных прямоугольников подвергается действию лапласовской силы под действием магнитного поля, создаваемого контуром. Эта сила перпендикулярна поперечной части и поэтому стремится отделить подвижную часть от остальной части схемы.

    Только расчет интенсивности силы, испытываемой подвижной частью, может помочь выбрать между двумя основными законами. Но можно ли вычислить, интегрируя формулу Ампера, силу отталкивания между двумя частями одной и той же цепи? Если сравнить проводник с линией без толщины, разделенной на «бесконечно короткие» токовые элементы, интегрирование приведет к бесконечной силе между двумя соседними коллинеарными элементами.

    Напротив, несколько авторов предложили методы расчета, которые лучше учитывают реальную структуру проводников: токовый элемент не является бесконечно коротким, а диаметр провода не равен нулю.

    — Для Грано размер элемента тока не может быть меньше, чем размер элементарной ячейки металлической решетки, порядка нанометра. Компьютеры выполнили численное интегрирование путем суммирования большого количества мелких элементов, но размер этих элементов остается намного больше нанометра.Путем экстраполяции полученных результатов можно надеяться получить величину силы порядка [Graneau, 1986].

    Рис. 4. Принцип расчета суммированием по конечным элементам [Graneau, 1986].
    Когда проводник делится на все меньшие и меньшие элементы, сила, вычисленная этим методом приближения, увеличивается, но все менее и менее быстро по мере приближения к атомным величинам.

    — Другие авторы, принимая во внимание объемное распределение тока в проводнике с ненулевым диаметром, избегают выбора численных методов и выполняют интегрирования.Следовательно, они должны иметь дело с шестикратными интегралами [Марсело Буэно и А. К. Т. Ассис, 1996].

    — Наконец, вычисления могут быть более легко выполнены в рамках классического электромагнетизма, используя выражение для индуктивности цепи, понятие, появившееся много позже Ампера [R.A.R. Tricker, 1965, или Bueno and Assis, 1998].

    Независимо от выбранного метода расчета сила, прикладываемая цепью к одному из ее элементов, имеет достаточно низкое значение, чтобы механические напряжения оставались незаметными в сплошных проводниках, по крайней мере, пока ток не имеет необычно высокой интенсивности.Но он может объяснить наблюдаемые эффекты в схемах с жидкостью или подвижной частью, как в предыдущих экспериментах.

    Некоторые авторы заканчивают триумфом формулы Ампера. По их мнению, интегрирования, основанные на формуле Грассмана, приводят к явно более слабым силам. Однако в 1996 году Ассис и Буэно исправили ошибку в рассуждениях, лежащих в основе этих вычислений, и достигли, используя формулу Грассмана, тех же результатов, что и формула Ампера. В 1998 году они также показали, что метод индуктивности представляет собой косвенное доказательство эквивалентности формул Ампера и Грассмана.

    Следовательно, не представляется возможным сделать выбор между формулами Ампера и Грассмана посредством экспериментов типа Ампера.

    Другие эксперименты вызывают споры

    В предыдущих экспериментах учитывалась общая сила, прилагаемая цепью к одной из ее частей. Хотя формулы Грассмана и Ампера дают одинаковый результат для этой общей силы, они могут не быть эквивалентными для распределения силы по контуру. Чтобы проверить эту гипотезу, некоторые пытались выделить локальных эффектов продольных сил, эффекты, которые могла объяснить только сила Ампера.

    — Вдохновленный экспериментом, проведенным Нейманом для его учеников в 1880-х годах, Грано поместил два коротких медных стержня встык в прямолинейный узкий канал, заполненный ртутью. Когда этот канал пропускал ток 450 ампер, два стержня отталкивались друг от друга.

    Рис. 5. В середине длинного канала ртути встык вставлены два медных стержня AB и CD. Когда течет ток, стержни отталкиваются друг от друга. (Стержни покрыты изоляцией, а проводимость меди намного выше, чем у ртути, что объясняет черный узор линий тока).Graneau, 1986.

    — В начале 1960-х Ян Насиловский уже заметил, что очень сильный ток вызывает взрыв проводящих проводов на серию фрагментов (рис. 6). Грано повторил эксперимент в 1980-х годах с токами более 5000 ампер, но с достаточно короткой продолжительностью, чтобы поддерживать температуру значительно ниже точки плавления металла. Исследование микроскопической структуры областей перелома, казалось, подтвердило механическую причину.Следовательно, переломы могли быть вызваны силой Ампера.

    Рис. 6. Осколки от взрыва проводов [Ян Насиловский].

    — В другом эксперименте [Линда Дж. Рускак и Р. Н. Брюс, 1987] медный стержень длиной один метр разрезали на сегменты по одному сантиметру и помещали в стеклянную трубку. Спрингс крепко держал их в контакте друг с другом. Импульсы тока от 3 до 30 килоампер вызвали их разделение, в то время как электрические дуги прыгали в промежутках.

    Рис. 7. Разделение и образование дуг между медными сегментами в эксперименте Рускака и Брюса.

    — Добавим эксперимент, проведенный после наблюдения парадоксального эффекта во время экспериментов с «электромагнитным рельсотроном» для Стратегической оборонной инициативы США. В левой части рисунка 8 показан очень простой принцип действия рейлгана или «пушки». Когда в цепи, состоящей из двух длинных горизонтальных рельсов и подвижного стержня, размещенного на этих рельсах, протекает очень сильный ток, стержень начинает двигаться.Придание движения снаряду очень классически объясняется силой Лапласа, испытываемой подвижным стержнем в собственном магнитном поле контура. В 1984 году при пиковой мощности более 2 миллионов ампер можно было вести объект весом 300 г со скоростью 15 000 км / ч.

    Во время этого эксперимента было замечено, что рельсы претерпели неожиданные механические деформации. Затем Грано изменил эксперимент (рис. 8, справа), заблокировав перекладину.Импульса в 100 килоампер было достаточно, чтобы вызвать деформации в самых тонких частях рельсов, которые он приписал продольным силам Ампера.

    Рис. 8: «Электромагнитный рельсотрон». В версии Грано (справа) перекладина заблокирована.

    Все эти эксперименты были представлены как доказательства существования продольных сил, но могут быть выдвинуты и альтернативные интерпретации.

    Другие эксперименты, направленные на демонстрацию продольных сил в жидких металлах и плазме. Хотя некоторые из них были очень драматичными, мы не будем упоминать их здесь, потому что они легче подвергаются альтернативным интерпретациям.

    Продольные силы и современный электромагнетизм

    Продольные силы в проводниках отсутствуют в большинстве книг по электромагнетизму. Кроме того, их потенциальные эффекты возникают только при необычно высокой силе тока. Тем не менее, по мнению некоторых авторов, электромагнетизм Максвелла не исключает существования продольных напряжений, которые могут играть роль в экспериментах, представленных выше.Эти напряжения возникают в результате применения к проводникам теории «тензоров напряжений» Максвелла, совершенно не связанной с силой Ампера.

    Сила Лоренца и сила Ампера: противоречие?

    Уравнения Максвелла имеют дело только с электромагнитным полем и не включают силовой закон. Для Максвелла формулы Грассмана и Ампера были совместимы с его теорией, хотя он предпочитал формулу Ампера.

    Позже Лоренц выразил силу, действующую на заряженную частицу q в вакууме при наличии магнитного поля:

    Сила Лоренца

    Отныне сила Лоренца, замечательно подтвержденная экспериментально, принята в классических описаниях электромагнетизма в качестве элементарного закона силы.В любой момент эта сила перпендикулярна направлению движения частицы. Таким образом, элемент тока, образованный движущейся заряженной частицей, испытывает силу, которая, как и сила Грассмана, не имеет продольной составляющей.

    Выражение силы Лапласа, , обычно представляется как следствие силы Лоренца. В моделях, предложенных для учета проводимости в металлах, все происходит так, как если бы силы, действующие на электроны проводимости, передавались металлической решетке.Поскольку сила перпендикулярна направлению движения электронов, металлический проводящий элемент может испытывать только поперечные силы. Таким образом, замечательная точность проверок закона Лоренца оправдала бы отказ от формулы Ампера.

    Тем не менее, оправдание закона Лапласа с помощью силы Лоренца было оспорено. Металлический проводник несравним с электронным пучком в вакууме. Мы должны учитывать физическую природу элемента тока, состоящего из положительных ионов и электронов.Согласно М. Рамбо и Ж. П. Вижье, применение законов электромагнетизма Максвелла-Лоренца-Эйнштейна к коллективному поведению этого ансамбля заряженных частиц приводит, в нерелятивистском приближении, к формуле Ампера (1989). Токи в металлах подчиняются закону Ампера, а токи в вакууме подчиняются закону Грассмана.

    Действие на расстоянии против теории поля и теории относительности Эйнштейна

    Тем не менее, хотя современный электромагнетизм предсказывает — и более простым способом — те же эффекты, что и формула Ампера, зачем спорить — как некоторые продолжают делать — за «реабилитацию» этой формулы? Есть ли еще повод для споров?

    Проблема на самом деле теоретическая.Сила Ампера — это сила мгновенного действия на расстоянии. В этом отношении это принципиально несовместимо с теорией Максвелла, основанной на непрерывном распространении электромагнитного воздействия.

    Триумф теории Максвелла основан на ее необычайной плодовитости. Его уравнения учитывают, среди прочего, распространение электромагнитных волн со скоростью c = 300 000 км / с. Электромагнитное поле, позволяющее переносить энергию, существует в вакууме, лишенном зарядов.Более того, именно теория Максвелла в некотором роде «заложила условия» для теории относительности Эйнштейна. Таким образом, здание электромагнетизма Максвелла-Лоренца и теории относительности Эйнштейна является, по-видимому, неоспоримым столпом всей физики двадцатого века.

    До Максвелла такие физики, как Вебер, а затем Нойман, разработали электродинамику, выведенную из формулы Ампера. Они искали закон силы или потенциала между элементами тока или между движущимися электрическими зарядами, который бы объяснил все явления, включая явление индукции, чего не делала формула Ампера.Таким образом, Вебер предложил элементарную силу между электрическими зарядами, состоящую из трех членов, зависящих от расстояния, скорости и ускорения одного заряда относительно другого. Первый член — не что иное, как закон Кулона, второй может быть выведен из формулы Ампера, а третий объясняет явление индукции.

    Некоторые современные защитники силы Ампера преследуют «веберовский» электродинамический проект как альтернативу проекту Максвелла (А.K. T. Assis, Weber’s Electrodynamics, 1994 или P. and N. Graneau, Newtonian Electrodynamics, 1996). Ссылка на Ньютона, как и ссылки на Ампера или его преемников, указывает на проект, выходящий за рамки области электричества. Это вопрос построения физики, основанной на взаимодействии на расстоянии между материальными элементами. Можно было бы возразить, что сами Ньютон или Ампер были далеки от того, чтобы рассматривать мгновенное действие на расстоянии как последнее слово в физике.За своими математическими законами гравитации или электродинамики, которые включают только взаимодействующие элементы и их расстояние, оба были убеждены в существовании более глубокой реальности, основанной на непрерывной передаче в промежуточном пространстве [см. Стр. Des théories mathématiquement équivalentes, Physiquement différentes ] . Однако это не мешает попыткам объяснить явления, обычно описываемые с помощью теории Максвелла, с использованием электродинамики Вебера: эффекты излучения, распространение света… «В этом направлении потребуется много теоретической и экспериментальной работы», — писал Ассис в 1989 году.

    Идея применения закона, аналогичного закону Вебера для электромагнетизма (включающего несколько терминов), к гравитации восходит к 1870-м годам и рассматривалась самим Вебером. К силе тяготения Ньютона добавляются некоторые члены, зависящие от относительной скорости масс, согласно модели терминов, добавленных Вебером к электрической силе Кулона. Согласно Ассису [ Relational Mechanics , 1999], эта веберовская механика позволяет вывести первые два закона динамики Ньютона — принцип инерции и фундаментальную взаимосвязь динамики — и установить пропорциональность между инертной и гравитационной массой. .Прецессию перигелия планет можно вычислить, не прибегая к уравнениям общей теории относительности …

    Широко распространенное инакомыслие, которое может вызвать подозрения?

    Когда изучаешь публикации некоторых сторонников восстановления силы Ампера, иногда можно обнаружить оппозицию господствующему истеблишменту — «официальной науке» , которая далеко не ограничивается электромагнетизмом Максвелла. Кто-то осуждает «аферу специальной теории относительности» или предлагает воспользоваться «бесконечной энергией вакуума».Другой противопоставляет стандартную теорию Большого взрыва теории стационарной Вселенной. Красный сдвиг света от далеких галактик, обычно интерпретируемый как расширение Вселенной, можно было бы объяснить с помощью теории «световой усталости». Также можно найти статьи о холодном синтезе и т. Д.

    Эти связи могут вызвать определенные подозрения. Однако можно вспомнить, что некоторые верования Ампера, не говоря уже о его интересе к животному магнетизму, пахли ересью для многих членов Академии.Конечно, вряд ли есть какие-либо экспериментальные основания сомневаться в электромагнетизме Максвелла, и может показаться, что работа над построением альтернативной электродинамики — пустая трата энергии. Тем не менее, нельзя ли одобрить рецензию Scientific American на книгу Грано, Newtonian Electrodynamics : «Настоятельно рекомендуется для тех, кто непредубежден, и всем, кто согласен с Максвеллом в отношении благоразумной роли плюрализма в обеспечении здоровья людей. наука и, собственно, обеспечение ее жизни.»?»

    Для получения дополнительной информации

    ГИЛЬЕМО, Элен. On avait oublié la force d’Ampère, Science & Vie , 879, 1990, p. 38-46.
    ASSIS, Андре; БУЭНО, М. Д. A. Расчет индуктивности и силы в электрических цепях . Нью-Йорк: Nova Science Publishers, Huntington, 2001.
    ASSIS, A. K.T. И ЧАЙБ, J.P.M.C. Электродинамический анализ Ампера значения и эволюции силы Ампера между элементами тока вместе с полным переводом его шедевра: Теория электродинамических явлений, уникально выведенная из опыта. Монреаль: Апейрон, 2015. [см. PDF]

    Библиография «вторичных источников» по ​​истории электричества.

    Французская версия: май 2009 г. (перевод на английский: март 2013 г .; последняя редакция: март 2021 г.)

    Электрические машины — генераторы и двигатели | Электродинамика

    11.2 Электрические машины — генераторы и двигатели (ESCQ4)

    Мы видели, что когда проводник перемещается в магнитном поле или когда перемещается магнит около проводника в проводнике течет ток.Величина тока зависит от:

    • скорость, с которой проводник испытывает изменяющееся магнитное поле,
    • количество витков, составляющих проводник, а
    • положение плоскости проводника относительно магнитного поле.
    Влияние ориентации проводника относительно магнитного поля проиллюстрирован на рисунке 11.1.

    Рисунок 11.1: Серия рисунков, показывающих, что магнитный поток, проходящий через проводник, зависит от от угла, который плоскость проводника составляет с магнитным полем.Величайший поток проходит через проводник, когда плоскость проводника перпендикулярна силовые линии магнитного поля, как на Рисунке 11.1 (а). Номер силовых линий, проходящих через проводник, уменьшается, так как проводник вращается до тех пор, пока он параллелен магнитному полю Рис. 11.1 (c).

    Если наведенная ЭДС и ток в проводнике были представлены как функция угла между плоскостью проводника и магнитным полем для проводника, имеющего постоянной скорости вращения, то наведенные ЭДС и ток будут варьируются, как показано на рисунке 11.2. Ток меняется около нуля. и известен как переменный ток (сокращенно AC).

    Рисунок 11.2: Изменение наведенной ЭДС и тока как угол между плоскостью проводника и проводником. магнитное поле изменяется.

    Угол изменяется как функция времени, поэтому приведенные выше графики могут быть нанесены на временную ось. также.

    Вспомните закон Фарадея, о котором вы узнали в 11 классе:

    Закон Фарадея

    ЭДС, \ (\ mathcal {E} \), индуцированная вокруг одиночной петли проводника, пропорциональна скорость изменения магнитного потока φ через площадь, \ (A \) петли.Математически это можно выразить как:

    \ [\ mathcal {E} = -N \ frac {\ Delta \ phi} {\ Delta t} \]

    , где \ (\ phi = B · A \ cos \ theta \) и \ (B \) — напряженность магнитного поля.

    Закон Фарадея связывает наведенную ЭДС со скоростью изменения магнитного потока, который является произведением напряженности магнитного поля и поперечного сечения область, через которую проходят силовые линии. Площадь поперечного сечения изменяется при вращении петли проводника. что дает фактор \ (\ cos \ theta \).\ (\ theta \) — угол между нормаль к поверхности витка проводника и магнитному полю. Когда проводник замкнутого контура меняет ориентацию по отношению к магнитному полю, величина магнитного потока, проходящего через область контура, изменяется, и в проводящем контуре индуцируется ЭДС.

    Электрогенераторы (ESCQ5)

    Генераторы переменного тока (ESCQ6)

    Используется принцип вращения проводника в магнитном поле для генерации тока. в электрических генераторах.Генератор преобразует механическую энергию (движение) в электрическую.

    Генератор

    Генератор — это устройство, преобразующее механическую энергию в электрическую.

    Схема простого генератора переменного тока показана на рисунке 11.3. Проводник представляет собой катушку с проволокой, помещенную в магнитное поле. В проводник вручную вращается в магнитном поле. Это порождает чередование ЭДС.Переменный ток нужно передать от проводника к нагрузке, это система, для функционирования которой требуется электрическая энергия.

    Нагрузка и проводник соединены контактным кольцом. Скользящее кольцо это соединитель, который может передавать электричество между вращающимися частями машины. Он состоит из кольца и щеток, одна из которых неподвижна. по отношению к другому. Здесь кольцо прикрепляется к проводнику и щеткам. прикреплены к нагрузке.Ток генерируется во вращающемся проводнике, проходит в контактные кольца, которые вращаются против щеток. Ток передается через щетки в нагрузку, и, таким образом, система получает питание.

    Рисунок 11.3: Схема генератора переменного тока.

    Направление тока меняется с каждой половиной оборота катушки. Когда одна сторона петли переходит в другую полюс магнитного поля, ток в контуре меняет направление.Этот тип тока, который меняет направление, известен как переменный. current, а на рис. 11.4 показано, как это происходит. как проводник вращается.

    Рисунок 11.4: Красные (сплошные) точки обозначают ток, исходящий со страницы, а крестики показывают текущий ток. переходя на страницу. Генераторы переменного тока

    также известны как генераторы переменного тока. Они используются в легковых автомобилях для зарядки автомобильного аккумулятора.

    Генератор постоянного тока (ESCQ7)

    Простой генератор постоянного тока устроен так же, как генератор переменного тока, за исключением того, что представляет собой одно контактное кольцо, которое разделено на две части, называемые коммутатором, поэтому ток в внешняя цепь не меняет направление.Схема генератора постоянного тока показана на Рисунок 11.5. Коммутатор с разъемным кольцом учитывает изменение направление тока в контуре, создавая тем самым постоянный ток (DC), проходящий через щетки и в цепь. Ток в петле меняет направление, но если вы посмотрите Внимательно изучив 2D-изображение, вы увидите, что секция коммутатора с разъемным кольцом также изменилась. какой стороны цепи он касается. Если ток меняет направление одновременно что коммутатор меняет местами стороны внешней цепи всегда будет иметь ток, идущий в в том же направлении.

    Рисунок 11.5: Схема генератора постоянного тока.

    Форма ЭДС от генератора постоянного тока показана на рисунке 11.6. ЭДС не является постоянной, но представляет собой абсолютное значение синусоидальной / косинусоидальной волны.

    Рисунок 11.6: Изменение ЭДС в генераторе постоянного тока.

    Генераторы переменного и постоянного тока (ESCQ8)

    Проблемы, связанные с замыканием и размыканием электрического контакта с движущейся катушкой, — это искрение и нагрев, особенно если генератор вращается с высокой скоростью.Если атмосфера, окружающая машину, содержит легковоспламеняющиеся или взрывоопасные пары, практические проблемы искрообразования щеточных контактов еще больше.

    Если вращается магнитное поле, а не катушка / проводник, тогда в генераторе переменного тока (генераторе) не нужны щетки, поэтому у генератора переменного тока не будет тех же проблем, что и у генераторов постоянного тока. Те же преимущества переменного по сравнению с постоянным током для конструкции генератора применимы и к электродвигателям. В то время как электродвигатели постоянного тока нуждаются в щетках для электрического контакта с движущимися катушками провода, электродвигатели переменного тока этого не делают.Фактически, конструкции двигателей переменного и постоянного тока очень похожи на их аналоги-генераторы. Электродвигатель переменного тока зависит от реверсивного магнитного поля, создаваемого переменным током через его неподвижные катушки с проволокой, заставляющими магнит вращаться. Двигатель постоянного тока зависит от замыкания и размыкания щеточных контактов. соединения для обратного тока через вращающуюся катушку каждые 1/2 оборота (180 градусов).

    Электродвигатели (ESCQ9)

    Основные принципы работы электродвигателя такие же, как и у генератора, за исключением того, что электродвигатель преобразует электрическую энергию в механическую энергию (движение).

    Электродвигатель

    Электродвигатель — это устройство, преобразующее электрическую энергию в механическую.

    Если поместить движущуюся заряженную частицу в магнитное поле, она испытал бы силу, называемую силой Лоренца .

    Сила Лоренца

    Сила Лоренца — это сила, испытываемая движущейся заряженной частицей в электрическом и магнитное поле.{-1} $} \)) и \ (B \) — напряженность магнитного поля (в теслах, Тл).

    На этой диаграмме показан положительный заряд, движущийся между двумя противоположными полюсами магнитов. В направление движения заряда указано оранжевой стрелкой. Он испытает Сила Лоренца, которая будет направлена ​​зеленой стрелкой.

    Токоведущий провод, в котором ток идет в направлении оранжевого стрелка, также будет испытывать магнитную силу, зеленая стрелка, из-за Лоренца сила на движущиеся отдельные заряды в текущем потоке.

    Если направление тока обратное для того же направления магнитного поля, то направление магнитной силы также будет обратным, как показано на этой диаграмме.

    Мы можем, если есть два параллельных проводника с током в противоположных направлениях. будут испытывать магнитные силы в противоположных направлениях.

    Электродвигатель работает за счет использования источника ЭДС, заставляя ток течь по петле проводник так, чтобы сила Лоренца на противоположных сторонах петли была противоположной направления, которые могут вызвать вращение петли вокруг центральной оси.

    Сила, действующая на проводник с током из-за магнитного поля, называется законом Ампера.

    Направление магнитной силы перпендикулярно обоим направлениям потока. тока и направления магнитного поля и можно найти используя Правило правой руки , как показано на рисунке ниже. Используйте ваш правая ; ваш первый палец указывает в сторону ток, второй палец по направлению магнитного поля и большой палец будет указывать в направлении силы.

    И двигатели, и генераторы можно объяснить с помощью катушки, вращающейся в магнитном поле. В генераторе катушка присоединена к внешней цепи, которая вращается, что приводит к изменению потока, вызывающему ЭДС. В двигателе катушка с током в магнитном поле испытывает силу с обеих сторон катушки, создавая крутящую силу (называемую крутящим моментом , , произносится как «разговор»), которая заставляет ее вращаться.

    Если используется переменный ток, для создания двигателя переменного тока требуются два контактных кольца.Двигатель переменного тока показан на рис. 11.7

    .

    Рисунок 11.7: Схема двигателя переменного тока.

    Если используется постоянный ток, для создания двигателя постоянного тока требуются коммутаторы с разъемным кольцом. Это показано на рисунке 11.8.

    Рисунок 11.8: Схема двигателя постоянного тока.

    Реальные приложения (ESCQB)

    Автомобили

    В автомобиле есть генератор. Когда двигатель автомобиля работает, Генератор заряжает аккумулятор и питает электрическую систему автомобиля.

    Генераторы

    Постарайтесь выяснить, какие значения тока вырабатываются генераторами переменного тока для разных типов машин. Сравните их, чтобы понять, какие числа имеют смысл в реальном мире. Вы найдете разные значения для автомобилей, грузовиков, автобусов, лодок и т. Д. Попытайтесь выяснить, какие другие машины могут иметь генераторы переменного тока.

    Автомобиль также содержит электродвигатель постоянного тока, стартер, который вращает двигатель и запускает его. Стартер состоит из очень мощного электродвигателя постоянного тока и соленоида стартера, прикрепленного к двигателю.Стартерному двигателю требуется очень большой ток для запуска двигателя, и он соединен с аккумулятором с помощью больших кабелей для передачи большого тока.

    Производство электроэнергии

    Для производства электроэнергии для массового распределения (в дома, офисы, фабрики и т. д.) обычно используются генераторы переменного тока. Электроэнергия, производимая массивными Электростанции обычно имеют низкое напряжение, которое преобразуется в высокое напряжение. это эффективнее распределять электроэнергию на большие расстояния в виде высоких напряжение в линиях электропередач.

    Затем высокое напряжение снижается до 240 В для потребления в домах и офисах. Этот обычно делается в пределах нескольких километров от того места, где он будет использоваться.

    Рисунок 11.9: Генераторы переменного тока используются на электростанциях (все типы, гидро- и угольные станции) для выработки электроэнергии.

    Высокие оценки в науке — залог вашего успеха и будущих планов. Проверьте себя и узнайте больше о практике Сиявулы.

    Зарегистрируйтесь и проверьте себя

    Генераторы и двигатели

    Упражнение 11.1

    Укажите разницу между генератором и двигателем.

    Электрический генератор — это механическое устройство для преобразования энергии источника в электрическую.

    Электродвигатель — это механическое устройство для преобразования электрической энергии из источника в энергию другого вида.

    Используйте закон Фарадея, чтобы объяснить, почему в катушке, вращающейся в магнитном поле, индуцируется ток.

    Закон Фарадея гласит, что изменяющийся магнитный поток может индуцировать ЭДС, когда катушка вращается в магнитном поле. Вращение может изменять магнитный поток, тем самым вызывая ЭДС.

    Если вращение катушки такое, что поток не меняется, т.е. поверхность катушки остается параллельно магнитному полю, то наведенной ЭДС не будет.

    Объясните основной принцип работы генератора переменного тока, в котором катушка механически вращается в магнитном поле.Нарисуйте диаграмму, подтверждающую ваш ответ.

    Решение пока недоступно

    Объясните, как работает генератор постоянного тока. Нарисуйте диаграмму, подтверждающую ваш ответ. Также опишите, чем генератор постоянного тока отличается от генератора переменного тока.

    Решение пока недоступно

    Объясните, почему катушка с током, помещенная в магнитное поле (но не параллельно полю), будет вращаться. Обратитесь к силе, действующей на движущиеся заряды со стороны магнитного поля и крутящего момента на катушке.

    Катушка с током в магнитном поле испытывает силу с обеих сторон катушки, параллельно магнитному полю, создавая крутящую силу (называемую крутящим моментом), которая заставляет его вращаться. Любая катушка, по которой проходит ток, может ощущать силу в магнитном поле. Сила обусловлена Магнитная составляющая силы Лоренца на движущихся зарядах в проводнике, называемая законом Ампера. Сила на противоположных сторонах катушки будет в противоположных направлениях, потому что заряды движется в противоположных направлениях.

    Объясните основной принцип работы электродвигателя. Нарисуйте диаграмму, подтверждающую ваш ответ.

    Решение пока недоступно

    Приведите примеры использования генераторов переменного и постоянного тока.

    Автомобили (как переменного, так и постоянного тока), производство электроэнергии (только переменного тока), везде, где требуется электропитание.

    Приведите примеры использования двигателей.

    Насосы, вентиляторы, бытовая техника, электроинструменты, бытовая техника, оргтехника.

    Насколько универсальны закон потока Фарадея и сила Лоренца?

    1 Насколько универсальны закон потока Фарадея и сила Лоренца? Резюме Герман Хертель Приглашенный научный сотрудник Института теоретической физики и астрофизики ITAP, Кильский университет Вопрос о том, является ли закон потока Фарадея универсальным или существуют исключения, долгое время оставался спорным.Это обсуждение, похоже, недавно пришло к выводу в пользу общности закона потока Фарадея. Настоящая статья снова поднимает этот вопрос с помощью некоторых довольно простых измерений, выполненных на генераторе Фарадея. Собранные результаты удивительны и требуют попытки согласовать их с предположительно общеприменимым законом потока Фарадея. Указана теория, альтернативная этому закону. Ключевые слова: электромагнитная индукция, закон потока Фарадея, сила Лоренца, основной закон электродинамики Вебера, генератор Фарадея.Введение Предмет «электромагнитная индукция» с двумя основными законами — «закон потока Фарадея» и «сила Лоренца» — как известно каждому опытному учителю — трудная тема для преподавания и понимания. Что касается преподавания, то в учебниках есть материалы по этой теме, которые часто подвергаются критике. Приемлемо ли понятие движущегося магнитного поля или мы должны обращаться к специальной теории относительности всякий раз, когда движущийся магнит находится в фокусе? Если рассматривать силу Лоренца как F = qe + q (vxb), будет ли v относительно поля, относительно лаборатории или наблюдателя? [Assis, Peixoto, 1992] Что касается понимания: какой механизм мог бы объяснить — хотя бы по аналогии — как взаимосвязь между изменением во времени магнитного потока по отношению к области и возникновением кольцевого электрического поля вокруг этой области устанавливается? Точно так же возникновение единственной силы без силы прямой реакции, действующей на носитель заряда, движущийся поперек магнитного поля, является единичным действием без ссылки на какой-либо другой известный процесс, который студенты могли бы использовать.Zuza et al. (2016) показали, что даже довольно хорошие студенты после завершения полного курса не преуспели, когда их попросили применить закон потока Фарадея или силу Лоренца к экспериментальным установкам, которые им еще не объяснили. Со стороны научного сообщества есть довольно много основных вопросов, по которым люди не согласны. Когда магнит вращается вокруг своих полярных осей как часть генератора Фарадея, вращается ли магнитное поле, везде постоянное по силе, вместе с магнитом или оно остается неподвижным независимо от вращения магнита? Келли [1998] опубликовал измерения, которые, как он утверждал, подтверждают магнитное поле, вращающееся вместе с вращающимся магнитом.Леус и Тейлор [2011] сделали тот же вывод

    2, основанный на своих собственных измерениях. Напротив, Chen et. al. [2017] на основе собственных измерений подтвердили, что магнитное поле не вращается вместе с магнитом, а остается неподвижным. Ассис и Тобер [1994] использовали совершенно другой подход и выбрали теорию Вебера в качестве основы для объяснения униполярной индукции. Существуют ли исключения из закона о потоках Фарадея или этот закон действителен при любых обстоятельствах? Фейнман [1969], аргументы которого приобрели репутацию, заявил, что существуют ситуации с изменением потока и без индукции, и наоборот.Галили и Каплан (1997) заявили, что иногда бывает проблематично использовать d-закон Фарадея в его интегральной форме = B. В качестве примера они dt B = B da A указывают на диск Фарадея, который вращается в постоянном однородном магнитном поле. поле. Такие заявления критикуют разные авторы [Scanlon et al, 1969], [Munley, 2004], [Zengel 2019]. Они утверждают, что происхождение исключений может быть связано с неправильным выбором пути тока. Скэнлон утверждает, что такой конфликт действительно никогда не может возникнуть из-за математического тождества между = E + v B и для данного контура.= d dt Манли очень подробно показывает, где Фейнман терпит неудачу, и заявляет, что закон Фарадея, правильно применяемый, может быть использован для вычисления наведенной ЭДС в любой ситуации, где может использоваться сила Лоренца. Зенгель выбрал довольно простую модель диска Фарадея, чтобы показать, как следует выбирать путь индуцированного тока и как следует применять закон Фарадея, чтобы показать его общность. Для этого при определении площади A, а также изменения магнитного потока d / dt по отношению к этой площади необходимо учитывать, что электроны индуцированного тока уносятся внутри движущегося тела и, таким образом, перемещаются на определенной территории.Если принять во внимание эту область, общая применимость закона потока Фарадея может быть доказана математически, возможно, путем преобразования во вращающуюся неинерциальную систему отсчета. Статья Зенгеля завершается следующим предложением: но ни один закон никогда не должен предсказывать результат, несовместимый с правилом потока: d = (dt B da) Это звучит так, как будто дискуссия об университете закона Фарадея наконец-то пришла к общепринятому решение. Следующее довольно простое измерение, проведенное на генераторе Фарадея, опровергает это утверждение.Фарадей обнаружил в 1832 году, что вращающийся магнит может функционировать как генератор постоянного тока. На рис. 1а показан такой генератор, состоящий из магнита и отдельного металлического диска, оба свободно вращаются вокруг полярной оси магнита, независимо или закрепленные. все вместе.

    3 w w S S a V Магнит из проводящего материала b V Рис. 1. Два варианта генератора Фарадея. Позже Фарадей обнаружил, что вращающийся магнит, состоящий из проводящего материала, может создавать наведенный постоянный ток (рис.1б). Следующие вопросы возникают с генератором Фарадея, который уже озадачил Фарадея: 1. Почему вы наблюдаете индукцию (как и ожидалось) с вращающимся диском и неподвижным магнитом, но не с обратным процессом: вращающийся магнит и неподвижный диск? 2. Почему наблюдается индукция, когда магнит и диск вращаются вместе? Эти вопросы вызвали длительные дискуссии. История началась с открытия Фарадеем парадоксальных результатов с его генератором в 1832 году; обсуждения продолжаются по сей день.Ключевыми вопросами в обсуждении являются те, которые уже упоминались выше: является ли магнитное поле вращающегося магнита стационарным или оно вращается вместе с магнитом? Закон потока Фарадея универсален или нет? Следующие ниже измерения выполняются либо на генераторе, как показано на рисунке 1b, либо в системе, в которой магнит и металлический диск вращаются вместе. Чтобы объяснить происхождение индуцированного тока в этих случаях, можно предположить, основываясь на классической теории, что магнитное поле остается стационарным и на него не влияет вращение магнита.Магнит, состоящий из проводящего материала, вращается через собственное магнитное поле. Свободные проводящие электроны разрезают силовые линии магнитного поля и ускоряются, в зависимости от ориентации магнитного поля, либо внутрь, либо снаружи магнита. То же самое объяснение можно применить и к металлическому диску, вращающемуся вместе с магнитом. В зависимости от направления магнитного поля обод и центр диска будут заряжаться в одну или противоположную сторону в зависимости от составляющей силы, задаваемой q (vxb).Измерение, вызывающее вопросы. Когда начинались измерения на генераторе Фарадея, первое контрольное измерение показало линейную зависимость измеренного индуцированного напряжения от скорости вращения (рис. 2).

    4 12 В (мВ) 9 В (мВ) Рис. 2 Наведенное напряжение V пропорционально скорости вращения. При проведении некоторых дополнительных измерений и использовании тонких скользящих контактов (тонких по сравнению с толщиной вращающегося диска) было замечено, что на показания явно влияет положение этих скользящих контактов на ободе вращающегося диска.Этот неожиданный эффект потребовал более детального изучения и привел к следующей экспериментальной установке (рис. 3): 20,0 мм Скорость вращения об / мин Скорость вращения w = 2620 об / мин A 6,8 мВ D Положение 4 30 мм 5 эодимных магнитов 9,2 мВ Положение 3 Нижняя часть гильзы SB 20,8 мм 6,0 мВ 7,4 мВ C Позиция 2 Позиция 1 Рис. 3 Установка для измерения наведенного напряжения на генераторе Фарадея, где диск и магнит вращаются вместе вокруг одной оси. Форма диска была расширена до втулки, так что индуцированное напряжение можно было измерять не только между осями вращения и дном втулки (соответствует ободу диска Фарадея), но также между различными точками на боковой стенке. рукава и оси вращения.Пружинные провода служили скользящими контактами для соединения проводников внешней цепи с вращающимися частями. Новизна этого эксперимента состоит в том, что диск Фарадея был расширен до гильзы, в которую были вставлены магниты, прочно соединены с нижней частью гильзы и электрически изолированы от гильзы.

    5 Боковые стенки вращающейся муфты в первом приближении движутся параллельно силовым линиям магнитного поля. Поэтому с классической точки зрения не ожидается, что эта вращающаяся втулка будет иметь какое-либо существенное влияние на результаты измерений.Более того, можно было ожидать, что такой же потенциал на ободе нижней части втулки будет существовать по всей металлической стенке втулки. Дело обстоит наоборот. Как показано на рисунке 3, показания увеличиваются более чем на 50%, когда скользящий контакт, начиная с обода диска (нижняя часть втулки), перемещается вдоль боковой поверхности втулки параллельно оси вращения ( позиции с 1 по 4). Показания достигают максимума на высоте центра магнита (положение 3) и после этого момента начинают уменьшаться.Возникает вопрос: как этот результат может быть совместим с законом потока Фарадея или силой Лоренца? Через магниты не проходят токи, так как они электрически изолированы от гильзы. При изменении точки измерения вдоль стенки рукава внизу ничего не меняется. Но мы получаем разные наведенные напряжения со скользящим контактом в разных положениях втулки. Ток протекает только по цепи ABCD (рис. 3). Где электроны, которые, согласно Зенгелю, проносятся по определенной области? Как эти электроны могут объяснить зависимость измеренного напряжения от положения скользящих контактов? Может быть, горизонтальные составляющие магнитного поля создают эффект, разрезая стенку рукава? Это маловероятно по двум причинам.С одной стороны, из-за тонкой стенки гильзы к ней прикасается только меньшая часть магнитного поля. Во-вторых, непонятно, почему возможное взаимодействие между магнитным полем и вращающейся стенкой гильзы должно давать максимальный сигнал на средней высоте магнита. При непредвзятом взгляде на достигнутые здесь результаты измерений кажется, что показания определяются не расстоянием между диском и магнитом, где пересекаются магнитные линии, а расстоянием между скользящими контактами. и магнит.Чтобы проверить эту идею, были повторены те же измерения с немного измененной настройкой. Магниты внутри гильзы теперь изолированы снизу проставкой из пластика высотой 5 мм. На рис. 4 показан результат, соответствующий вышеупомянутой идее: кажется, расстояние между скользящим контактом и магнитом определяет показания.

    6 Скорость вращения w = 2620 об / мин 5 эодимных магнитов 9,2 мВ 5 эодимных магнитов S 6,0 мВ Промежуточный блок 2,0 мВ Обсуждение Рис.4 Изменена установка по сравнению с рис. 3 за счет изоляции магнита от нижней части втулки с помощью распорного блока. Схема, показанная на рисунках 3 и 4, слишком проста, а результаты измерений слишком поразительны, чтобы ими можно было пренебречь как грязный эффект. Последствия далеко идущие. Придется отказаться от идеи стационарного магнитного поля вокруг вращающегося магнита. Силовые линии магнитного поля, пересекающие диск Фарадея, не вызывают индукции. d Измерения не соответствуют правилу магнитного потока = (как того требует Зенгель dt B da).Расстояние между магнитом и скользящим контактом очень важно. Вкратце: предположение об общей справедливости закона потока Фарадея и силы Лоренца сомнительно. В свете альтернативной теории, а именно Основного закона электродинамики Вильгельма Вебера, эти следствия становятся еще более очевидными. Теория Вебера четко объясняет все эти измерения. Согласно этой теории [Weber, 1846], [Assis, 1994] важными членами являются относительная скорость и относительное ускорение между взаимодействующими зарядами.Всякий раз, когда происходит относительное движение между двумя отдельными носителями заряда, всегда существующее кулоновское взаимодействие между этими частицами незначительно изменяется с коэффициентом, пропорциональным 1 / c 2 (c = скорость света). Если исследуются два нормальных макроскопических объекта, все возможные кулоновские взаимодействия уравновешиваются тем фактом, что материя нейтральна, имеется равное количество положительных и отрицательных частиц. Если относительное движение между положительными и отрицательными частями нейтрального тела различно, это равновесие будет нарушено, и фактор Вебера, пропорциональный 1 / c 2, станет видимым в форме магнитных сил.

    7 В теории Вебера существуют прямые силы, действующие вдоль прямой, соединяющей взаимодействующие частицы, F = q (vxB) отсутствует. В установке, описанной в этой статье, взаимодействующие носители заряда, которые движутся относительно друг друга, представляют собой отрицательные проводящие электроны внутри скользящего контакта (в состоянии покоя относительно лаборатории) и заряды как часть молекулярных токов 1 внутри магнита, вызывающих его магнетизм. Между положительными частями нет относительных движений.Следовательно, из-за фактора Вебера кулоновские силы больше не уравновешивают друг друга, и возникает ЭДС. Когда скользящий контакт перемещается вверх от нижней части втулки (рис. 2 и 4), расстояние между центром магнита и скользящим контактом уменьшается. Поскольку кулоновские силы зависят от расстояния между взаимодействующими партнерами, то же самое зависит и от индуцированной ЭДС. До сих пор все известные индукционные процессы феноменологически объяснялись теорией Вебера [Haertel, 2018]. Дополнительную информацию можно найти в литературе [Assis, 1994].Сообщалось о различных экспериментах, результаты которых согласовывались с теорией Вебера. Уэсли [1972] рассчитал поведение Z-антенны и нашел согласие между теорией и экспериментом только на основе теории Вебера. Гуала Вальверде [2002] разработал сложную экспериментальную установку, чтобы показать в соответствии с Вебером, что между магнитом и внешней цепью должна существовать относительная скорость, чтобы вызвать индукцию. Стоит обратить внимание на результаты эксперимента Келли [1998], в котором говорится, что магнитное поле действительно вращается вместе с магнитом.Это противоречит стандартному объяснению вращающегося магнита, прорезающего собственное стационарное поле. Это косвенно поддерживает подход Вебера, который требует относительного движения между магнитом и внешней цепью. Дидактические соображения Помимо нескольких случаев, два закона, закон потока Фарадея и закон силы Вебера, приходят к одному и тому же окончательному выводу при объяснении электромагнитных и электродинамических явлений. История физики и техники, насчитывающая почти 200 лет, показала, что закона Фарадея достаточно, чтобы поддержать это развитие как основной закон природы.Почему необходимо или иметь смысл иметь дело со второй теорией? Главный аргумент в пользу теории Вебера — дидактический. Тема «электромагнитная индукция» есть практически в каждом учебнике физики. Однако, как указывалось выше, это сложная тема для преподавания и тема с плохими результатами обучения (Zuza et all, 2016). Многие учителя довольны, если их ученики могут успешно применять правило правой руки. Невозможно достичь более глубокого понимания причин, по которым изменяющийся магнитный поток вызывает круговое электрическое поле.То же самое относится к силе Лоренца и к следующему вопросу: почему 1. Понятие молекулярных токов было введено Ампером.

    8 есть сила без силы прямой реакции? Когда студенты терпят неудачу, пытаясь достичь более глубокого понимания, они могут прийти к выводу, что физика слишком сложна, и могут потерять интерес к этой теме. Или они могут прийти к выводу, что их интеллектуальных способностей недостаточно для изучения и понимания физики.Существует риск того, что эти студенты могут отказаться от дальнейшего обучения. Если бы не было альтернативы закону Фарадея, эту ситуацию пришлось бы принять, и такие студенты могли бы потеряться для физики. Однако теория Вебера является такой альтернативой. Он объясняет явления за счет уменьшения сложности и гораздо больше подходит для более глубокого понимания. Есть, например, только силы притяжения или отталкивания, действующие вдоль линии соединения взаимодействующих частиц.Более того, в этой теории нет векторного произведения vxb. С дидактической точки зрения игнорировать такую ​​альтернативу недопустимо. Дополнение В заключение следует сказать о Вильгельме Вебере, который в значительной степени неизвестен и чья теория часто уничижительно описывается, что это только теория действия на расстоянии, а не теория поля. Верно, что теория Вебера — это теория действия на расстоянии, как и закон Кулона или универсальный закон всемирного тяготения. То, что широко не известно, — это тот факт, что Вебер первым установил телеграфное уравнение.Вебер не только теоретически предсказал возможность телеграфа, но и реализовал его, протянув два провода над крышами Геттингена, чтобы соединить свой Физический институт с Астрономической обсерваторией под руководством К. Ф. Гаусса, через которую он мог посылать электрические импульсы. В память об этом событии в Геттингене есть мемориал. На нем изображены Гаусс и Вебер, и он призван ознаменовать момент изобретения и внедрения первого телеграфа. (Литературный Ассис, А.К.Т. и Пейшото Ф. М. (1992), О скорости в законе силы Лоренца, Учитель физики, Vol. 30, pp Assis, AKT и Thober, DS (1994), Unipolar Induction and Weber’s Electrodynamics, in Frontiers of Fundamental Physics, M. Barone и F. Selleri (ред.), Plenum Press, ew York, pp Assis, AKT (1994) ), Электродинамика Вебера (Клувер, Дордрехт). В настоящее время эта книга доступна через Springer. Чен К., X.-J. Ли и Ю.-Х. Хуэй (2017), Экспериментальное исследование униполярной индукции, Acta Physica Polonica A, Vol.131, стр. Фарадей, М. (1832), Философские труды Лондонского королевского общества, Vol. 122, стр. Feynman, R.P .; Лейтон, Р. Б. и Сэндс, М. (1969), Лекции Фейнмана по физике, электромагнетизму I, Аддисон-Уэстли, 17 / 3-4.

    9 Галили И. и Каплан Д. (1997), Изменение подхода к обучению электромагнетизму в концептуально ориентированном вводном курсе физики, Am. J. Phys. 65, 657. Хэртель, Х. (2018), Электромагнитная индукция: альтернатива для обучения и понимания.Европейский журнал физического образования, 9, 2, стр. Гуала-Вальвердеа, Дж. Маццони, П. и Ахиллес, Р. (2002), Униполярный двигатель: настоящий релятивистский двигатель, Am. J. Phys. 70, Келли А.Г. (1998), Последняя загадка Фарадея: вращается ли поле с помощью магнита? Монографии 5 и 6 Института инженеров Ирландии. Леус В. и Тейлор Св. (2011), О движении поля постоянного магнита, Eur. J. Phys. 32, Манли, Ф. (2004), Проблемы правила потока Фарадея, Американский журнал физики 72, Scanlon, P.J .; Хенриксен Р. и Аллен Дж. Р. (1969), Подходы к электромагнитной индукции, Американский журнал физики, том 37, 698.

    alexxlab

    Добавить комментарий

    Ваш адрес email не будет опубликован. Обязательные поля помечены *